Lsat
Lsat
A. 9:30 or 10:30 If Olivia sits next to Jack, then she doesn't sit next to Lee.
B. 9:30 or 10:00
C. 10:00 or 11:00 7) Of the following, which one is a possible seating
D. 10:00 or 11:30 arrangement of the six friends?
E. 9:30 or 11:30
A. Kimberly, Nick, Jack, Olivia, Melissa, Lee
2) Which one of the following is a possible ordering of the 5 B. Lee, Melissa, Kimberly, Jack, Olivia, Nick
people's appointments? C. Melissa, Lee, Jack, Nick, Kimberly, Olivia
D. Lee, Melissa, Jack, Nick, Kimberly, Olivia
A. Sarah, Molly, Nick, Eric, Bill E. Lee, Olivia, Jack, Kimberly, Nick, Melissa
B. Bill, Sarah, Molly, Nick, Eric
C. Sarah, Molly, Bill, Nick, Eric 8) If Jack sits next to Kimberly, then which one of the
D. Sarah, Molly, Nick, Bill, Eric following pairs of people must be seated next to each other?
E. Molly, Nick, Bill, Sarah, Eric
A. Lee and Nick
3) If Sarah's appointment is at 10:30, which one of the B. Melissa and Nick
following must be false? C. Jack and Nick
D. Melissa and Kimberly
A. Bill's appointment is at 11:00 E. Olivia and Kimberly
B. Sarah's appointment is before Nick's
C. Nick's appointment is before Sarah's 9) If Jack and Olivia sit next to each other, then Lee
D. Bill's appointment is after Sarah's CANNOT sit next to both:
E. Molly's appointment is at 9:30
A. Jack and Nick
4) If Bill does not show up for his appointment, which B. Jack and Kimberly
possible appointment slot has opened up for Katie to take? C. Nick and Melissa
D. Jack and Melissa
A. 9:30 or 11:00 E. Kimberly and Nick
B. 10:00 or 10:30
C. 10:00 or 11:00 10) If Jack sits next to Olivia, then which of the following is a
D. 10:30 or 11:00 complete and accurate list of the people who could also sit
E. 10:00 or 11:30 next to Jack?
A. Melissa sits next to Nick and Lee. 9:30 10:00 10:30 11:00 11:30
B. Nick sits next to Kimberly and Lee. Sarah Molly Nick Bill Eric
C. Olivia sits next to Nick and Jack.
D. Lee sits next to Olivia and Jack. Diagram #3
E. Melissa sits next to Jack and Lee.
9:30 10:00 10:30 11:00 11:30
Answers Sarah Bill Molly Nick Eric
1. There are five time slots for the appointments at 9:30, 10:00, 1) E. Looking at the three diagrams, Nick can have an
10:30, 11:00, and 11:30. appointment at 10:00, 10:30, or 11:00. So 9:30 and 11:30 or E is
the correct answer.
10:00 10:30 11:00 11:30
2) D. This is solved by simply comparing each choice to the
First, the conditions should be diagramed. Start with the most diagrams until one matches. A is incorrect since Eric isn't in the
restrictive condition and build from there. This is condition #3; 11:30 slot, B and E are incorrect since Sarah can't be in the 10:00
Eric must have the 11:30 appointment slot. or 11:00 slots, and C is incorrect since Nick must come
immediately after Molly. D is the correct answer as it matches
the second diagram.
10:00 10:30 11:00 11:30
Eric
3) B. Since the question specifies that Sarah's appointment is at
10:30, the first diagram is the one to use. While each answer
The second most restrictive is condition #2; Sarah cannot have could be compared the diagram until one that doesn't match is
the 10:00 or 11:00 appointment slot. Since Eric is already in the found, the easier way is to notice that B and C are exact
11:30 slot, this leaves two possible slots for Sarah. opposites and so one of them must be the correct answer.
Looking at the first diagram, Nick's appointment comes before
10:00 10:30 11:00 11:30
Sarah's making B false and thus the correct answer.
Eric
10:00 10:30 11:00 4) C. Condition #1 states that Katie only gets an appointment if
11:30
Sarah Ericone of the others cancels, so since Bill has canceled this means
that is slot is open. Looking at the diagrams for where Bill's
appointment is, this means that either 10:00 or 11:00 has opened
Next we add condition #4; Molly's appointment slot is
immediately before Nick's. If Sarah has the 10:30 appointment, up and that C is the correct answer.
then there is only one place Molly and Nick's appointments can
be. 5) D. Look at each of the three diagrams and find the one where
Molly's appointment is at 10:00, which is the second diagram. In
the second diagram Bill's appointment is at 11:00 making D the
10:00 10:30 11:00 11:30
correct answer.
Nick Sarah Eric
6) E. This one actually changes one of the conditions and
However, if Sarah has the 9:30 then there are two possible places requires a new diagram. Eric's appointment is moved from 11:30
for Molly and Nick's appointments. to 11:00.
This is easy since the diagram from answer #2 shows that Kimberly
can sit next to Jack in this scenario making D the correct answer.
Or simply: 11) A. This one requires a diagram before any answer choices
can be eliminated. First, place Jack and Nick next to each other
on the diagram and Kimberly on the other side of Nick due to
condition #3 - KN. Lee must sit on the other side of Jack, since
condition #1, ~(MJ), prevents Melissa from sitting there and
condition #2, JO e/or JN, prevents Olivia from sitting there.
Olivia and Melissa cannot be placed on the diagram as they
could each go in either seat.
7) C. Simply compare each answer choice to the conditions; A
breaks JO e/or JN, B breaks KN, D breaks ~(MJ), and E breaks
OJ -> ~(OL). Only C doesn't break any of the conditions making
it the correct answer.
A. believes that his loyalty to the company will guarantee Miteki: Kauai has a traditional commitment to environmental
that he keeps his job integrity. The one-lane bridges were installed to minimize the
B. does not realize that the company is already bankrupt impact on the island environment. To widen the one-lane bridges
and has to lay off many employees could have a dangerous effect on the native plant and animal life.
C. equates his past success with future opportunity
regardless of the economic conditions
As a response to Lito's argument, Miteki's comment is should restrict accommodation choices to three- and
flawed because she four-star resorts.
C. Research has shown that traders who put their money in
A. relies on faulty information to support her argument options tend to be more likely to suffer from losing
B. uses circular reasoning to make her main points trades. Research also suggests that traders who put their
C. focuses on a minor issue instead of a more important money in stocks, which require a larger investment up
one front, tend to be more successful. Therefore, traders
D. responds to a supporting point instead of a main point should put their money into stocks if they hope to have
E. fails to address the substance of Lito's claim by making winning trades.
a secondary argument D. Studies suggest that students who are most successful in
college have attended advanced placement classes in
high school. Therefore, those students who hope to be
6. Art scholar Herbert Read has noted of Impressionist artist
most successful in college should strive to be accepted
Pierre-Auguste Renoir that he was the final painter
into advanced placement classes.
representing the artistic tradition that started at Rubens and
E. Caitlyn and Moira discovered that Caitlyn was spending
ended at Watteau. Peter Paul Rubens, the Flemish artist who
a little more than Moira at the grocery story each
flourished in the seventeenth century, is renowned for his
month. But Moira usually stopped at the grocery store
creative choice of subject matter, from landscapes to
several times each week, while Caitlyn never shopped
allegory. Antoine Watteau, the French-born painter who
more than once a week. Therefore, Caitlyn and Moira
died early in the eighteenth century, is generally
are actually spending the same amount, because Moira
remembered for his ability to interweave themes from Italian
must pay the extra cost of gas for several trips to the
theatre into his paintings. During the late nineteenth century
store.
in which he thrived, the French artist Renoir was
particularly renowned for his application of light and
shading, his use of vibrant color, and his ability to create an 8. The CEO of a major fast-food chain just released a
intimate scene. statement indicating that the chain has revamped its
ingredient list and revised its menu options so that all
Which of the following best summarizes the main conclusion selections on the menu are now healthier than any of the
of the passage above? menus from competing fast-food chains. Among the changes
that will be made, the fast-food chain will be eliminating
unsaturated fats and downsizing portions. He claims, "We've
A. Like Rubens and Watteau, Renoir selected subject heard the request of our customers for healthier meals, and
matter that was creative and theatrical. with the changes we're making we'll be offering our
B. Read believes that Rubens, Watteau, and Renoir are customers better choices. Our customers will now have
among the greatest painters in Western history. healthy fast-food options that put all other fast-food chains to
C. Read believes that apart from Renoir, no great artist of shame."
note has arisen since the early eighteenth century.
D. Relative to other painters of Renoir's era and later ones,
Renoir best carried on the tradition of individual style Which of the following, if true, most seriously undermines
that defined earlier artists. the CEO's claim of offering healthier selections to
E. The painters who had the greatest impact between the customers?
seventeenth and late nineteenth centuries were either
Flemish or French. A. While the fast-food chain is improving certain
ingredients, they are still including additives that tend to
make customers addicted to the fast food.
7. School Principal: Recent testing indicates that students in
B. The CEO is receiving a large bonus for the potential
our school are struggling in math. At this time, the students
boost in sales that the new menu options are expected to
are spending only forty-five minutes each day on math
bring.
lessons. The testing also indicates that students are excelling
C. A focus group unanimously complained that the food
in reading, however. The students currently spend one and a
was bland and not as tasty as items on the previous
half hours each day on reading. Therefore, we need to
menu.
increase the amount of time spent on math in order to
D. The fast-food chain has begun marketing heavily to
improve the math skills of students in our school.
children, encouraging children to ask for the healthier
options when ordering in a restaurant.
The weakness in the school principal's argument is similar to E. The fast-food chain has not removed many of its
the weakness in which of the following arguments? previous menu items but has instead simply replaced
the unhealthy ingredients with healthier options and
A. Family psychologists have found that the children with reduced the portion sizes.
the best speech development are those whose parents
read to them at an early age. Therefore, all parents 9. Head of a regional psychiatric association: As an
should read to their young children in order to enhance organization, we have found that patients over the last two
their speech development skills. decades are increasingly likely to suffer from depression that
B. Travel agents routinely see that families who have the in some cases can lead to an early death. At the same time,
most comfortable and relaxing vacations are those who we have found that those patients who already practice some
stay in three- or four-star resorts. Therefore, any family form of spirituality tend to be less likely to suffer from
hoping to have a comfortable and relaxing vacation depression and thus to live longer and healthier lives. A
recent study in a major psychological journal confirms this struggling with obesity to lose weight and that this method of
experience. As a result, we suggest that our members begin losing weight is the best method. The student must consider each
encouraging their patients to explore spirituality in the hopes answer choice in the context of what the nutritionist says in order
that it will provide them with longer and healthier lives. to determine which answer is most correct.
The primary argument made by the head of the regional The Correct Answer:
psychological association depends on which of the following
assumptions? B: The nutritionist's statement that a healthy, balanced diet and
the incorporation of daily exercise is strongly undermined by the
A. All of the patients being treated by psychiatrists in the information that significant factors in weight loss include
regional psychiatric association are suffering from physical makeup and metabolism, and that the nutritionist
depression. worked only with adults with very similar physical makeups and
B. All patients noted for their longer and healthier lives metabolisms. This information throws into question whether the
were practicing the same form of spirituality. nutritionist's methods would work for adults with different
C. All members of the psychiatric association must also physical makeups and metabolism, thereby undermining the
pursue spirituality in order to make educated claim the nutritionist's method for weight loss is the best method.
recommendations to their patients. Therefore, answer choice (B) is the correct answer.
D. When untreated, depression is a serious condition that
leads to death. The Incorrect Answers:
E. All forms of spirituality are equally healthy, and any
form of spirituality will provide patients with longer A: Answer choice (A), although tempting, is ultimately
and healthier lives. irrelevant to the nutritionist's direct claims about the best method
for weight loss; answer A is considered an ad hominem claim,
10. Economists have noted in recent weeks that the price per that is, a claim directed about a person making an argument
barrel of crude oil has decreased sharply over the last few rather than about the merits of the argument itself. Simply
months, dropping as much as seventy percent. They have because the nutritionist receives funding from the government
also found, however, that the price of gasoline at the pump does not mean that the nutritionist's study is biased. Nothing in
has not seen a similarly sharp reduction, and gasoline prices the passage suggests that it is. Answer choice (A) is not correct.
have dropped only about fifty percent.
C: Like answer choice (A), answer choice (C) is interesting but
Given the statements above, which of the following most does nothing to undermine the nutritionist's claims. It is not
helps to explain the difference between the drop in the price surprising to learn that another nutritionist disagrees with him,
of crude oil per barrel and the drop in the price of gasoline at but disagreement with a colleague does not necessarily
the pump? undermine his test results. Answer choice (C), therefore, is
incorrect.
A. The demand for crude oil worldwide has suddenly
decreased; this has led to a drop in the price of crude D: Far from undermining the test results and the nutritionist's
oil. claims, answer choice (D) actually supports them. This is
B. While crude oil prices were high, refineries that because different methods of exercise might have different
processed crude oil into gasoline absorbed a large part results regarding weight loss. Thus answer choice (D) can be
of the cost; these refineries are now recovering some eliminated at once.
profit by not yet passing the decreased price in crude oil
to customers at the pump.
E: The statement about the number of patients provides further
C. Oil companies have recently discovered a large and
information about how the nutritionist carried out his test.
previously untapped oil reserve; this discovery
However, it does not necessarily undermine the results. Eighty
immediately sent crude oil prices plummeting.
patients could conceivably provide the nutritionist with a large
D. A major wind energy company unexpectedly enough study group to derive useful results, so this choice does
announced plans to provide a large-scale alternative
not necessarily undermine his claims. Answer choice (E) is
energy option to citizens in several nations; this created
incorrect.
a competition for crude oil that negatively affected its
price.
E. Due to the increasingly low cost of crude oil, the cost of Question 2
production now exceeds the return in value; this has
forced some oil production companies to go out of Overview: Question 2 presents a scenario in which someone
business. (Mike) is attempting to predict future events on the basis of his
past experience. Three important factors must be considered: (1)
Logical Reasoning Answers the company for which Mike works is suffering due to the
economic downturn, (2) the detail that Mike has had a strong
sales record in the past but has struggled to maintain it in recent
Question 1 months because of the weak economy, and (3) the fact that
Mike's boss has asked to see him. Mike concludes that his boss
Overview: This question asks students to select the statement willnot fire him because of his time with the company, his
that most seriouslyundermines the nutritionist's claims that his excellent record, and his potential to succeed in spite of the
method of applying diet and exercise alone helped patients
crisis. The student is asked to consider how Mike's reasoning is Most of the medieval music still in existence is sacred music." If
flawed. most extant medieval music is sacred music, and any music not
recorded on manuscripts has been lost to history, then it follows
The Correct Answer: that most popular music was not recorded on manuscripts and
thus has been lost to history. It can safely be inferred from this
C: Answer choice (C) is the only selection that takes all of the fact that historians do not know much about popular medieval
music because they have almost nothing to study. Answer choice
elements into account and summarizes the substance of the
(E) is correct.
problem with Mike's argument: he believes his success in the
past will translate automatically to future success, despite his
failure over the last few months to keep his sales high. He also The Incorrect Answers:
believes that the company will look only at his past success and
will not consider current factors, including his recent struggles A: Answer choice (A) is incorrect because the passage does not
and the economic situation as a whole. Thus answer choice (C) is offer an evaluative judgment on the music that is still extant, nor
correct. does it suggest anywhere that sacred music was recorded
because it was the "greatest music" of the period. Historians
The Incorrect Answers: might rightly debate this issue, but the passage does not discuss
it or imply anything about it.
A, E: Although answer choices (A) and (E) do address separate
parts of the problem with Mike's reasoning, neither addresses all B: As with answer choice (A), the passage does not imply
of it. Mike's belief in his loyalty to the company ("I've been answer choice (B). The passage describes facts about the way
working with the company for well over a decade") and in the medieval music was recorded and about what medieval music
company's confidence in his ability to overcome the tough still exists, but it does not provide an answer to why most of the
economy ("If anyone can find a way to boost sales and benefit extant music from the Middle Ages is sacred music. Therefore,
the company, I'm the person") both contribute to the flaw in his in no place in the passage does the author suggest that the
argument, but neither encompasses it fully. Both answer choices Church recorded only medieval music because it did not value
(A) and (E), therefore, are incorrect. popular music. Answer (B) should be eliminated immediately.
B: Answer choice (B) provides an interesting piece of C: While answer choice (C) might very well be true, the
information, but in terms of Mike's argument it is largely substance of this statement is not discussed at all in the passage
irrelevant and does not address the flaw in Mike's reasoning in and cannot be inferred from any statement made within it.
any way. It contributes a piece of information that might support Answer (C) is thus irrelevant and should be eliminated
the theory that the company will indeed fire Mike, but it fails to immediately.
explain how Mike's own argument is problematic. D Answer
(D), again, is interesting but irrelevant to Mike's argument and D: The passage does indicate that the Church was not necessarily
does not address the flaw in his reasoning. That Mike is a close the only institution to have the means of affording manuscripts:
friend of his boss might make it more difficult for his boss to fire "As a result, few were able to produce or own them, and the
him, but since Mike does not mention this in his statement in any Catholic Church, which had literate scribes as well as
way, it cannot be assumed that considerable wealth, produced and maintained most manuscripts
during the Middle Ages." This suggests logically that other
Questions 3 and 4 institutions such as the aristocracy might very well have been
able to produce and maintain some of the manuscripts. But
Overview: Questions 3 and 4 regard a passage about the nowhere does the passage discuss the contents of aristocratic
households or that aristocrats might have held large numbers of
manuscript history of medieval music, noting specifically four
manuscripts, so this statement cannot be inferred from the
important facts: (1) manuscripts were expensive to produce, (2)
passage.
the Catholic Church was one of the few institutions able to take
on this expense, because it had both the wealth and the human
resources as represented by scribes, (3) most extant music from Question 4
the medieval period has been recorded on manuscripts, and (4)
the majority of this music is sacred music. The passage includes The Correct Answer:
the added suggestion that most popular music from that era is
unknown today because very little was recorded on manuscripts. A: In question 4, the student is asked to consider which
statement cannot be inferred from the passage. The passage
Question 3 makes a number of claims about manuscripts in the medieval
period, but the only comment on the parchment from which the
The Correct Answer: manuscripts were made is that it was expensive. It might seem
logical to argue that if the parchment was expensive, it must also
have been difficult to produce, but there is no statement in the
E: In question 3, the student is asked to select a statement that is
passage that supports this claim in any way, so the conclusion
supported by the claims made in the passage. To do this, the
cannot be drawn safely. Therefore, answer choice (A) is correct.
student must infer from what is stated directly. Based on these
claims, answer choice (E) is the only one that fulfills this
requirement. The passage states quite clearly, "Any medieval The Incorrect Answers:
music not recorded on manuscripts has now been lost to history.
B: The passage notes that the manuscripts were "painstakingly A: The conversation does not discuss the accuracy of the
copied by hand" and that the only people qualified to do this information in either Lito's argument or Miteki's, so there is no
work were the scribes (who were literate), suggesting that the way to know whether Miteki is relying on faulty information.
work required a great deal of time and effort and that few people Answer choice (A) may be eliminated immediately.
were able to copy manuscripts. From this, answer choice (B)
may be inferred from the passage and is thus incorrect. B: Although Miteki's response does indeed fail to address the
substance of Lito's argument, her mistake is not that of circular
C: The passage claims that the Church produced most of the reasoning, so answer choice (B) may also be eliminated
manuscripts. The passage also indicates that the majority of immediately.
extant medieval music is sacred (i.e., deriving from the Church),
so it may be inferred that the Church was selective about what C: There is no suggestion in the conversation that either topic is
music was copied down. Therefore, the passage does imply the more important than the other, so the judgment that the
statement in answer choice (C). (Note that this does not substance of Miteki's response is less important than Lito's
necessarily suggest that the Church did not value popular music - cannot be inferred from the conversation. Moreover, such a
see answer choice (B) from question 3 - but that it was selective. judgment does not address the flaw in Miteki's reasoning, so
The two qualities must be distinguished from one another.) answer choice (C) cannot be correct.
D: The passage notes that the manuscripts were "copied by D: Miteki responds to Lito by picking up on one element of his
hand" and that the Church had "literate scribes" to do this job, argument - the one-lane bridges - and then developing an
suggesting that within the Church they alone were qualified to unrelated argument of her own. She does not, however, develop
copy down manuscripts. As a result, the passage does imply that any of Lito's supporting claims (i.e., that the number of accidents
only the literate (those who know how to read) were allowed to has increased, that there are more tourists in Kauai), so answer
copy down manuscripts. Note that the question does not say that choice (D) is incorrect.
of all members of society, the scribes alone were allowed to copy
down manuscripts. It is entirely possible that literate members of
Question 6
the aristocracy copied down manuscripts. What is significant
with regard to this question is that scribes were literate, not that
they were members of a certain class within society. Overview: Question 6 offers a quotation from an art scholar
about the Impressionist artist Renoir, in which Renoir is
discussed within the context of two earlier artists. The passage
E: The passage comments, "Any medieval music not recorded
offers details about these earlier artists, as well as information
on manuscripts has now been lost to history. Most of the about Renoir's personal artistic style. The question asks for a
medieval music still in existence is sacred music." From this it statement that best summarizes the conclusion of the passage.
may rightly be inferred that most non-sacred music from the
Because the passage does not state its main point directly, the
medieval period has been lost to history, so the passage implies
student must infer that statement of summary from the
answer choice (E), and answer choice (E) is incorrect.
information within the passage.
E: In her response, Miteki ignores the substance of Lito's The Incorrect Answers:
argument - the potential danger from the one-lane bridges and
the need to widen them - and instead tries to redirect the
A: The only artist noted specifically for his subject matter is
conversation toward a related but different topic, specifically the Rubens, and the passage does not indicate that this quality may
environmental impact of widening the bridges. Thus, answer be attributed either to Watteau or Renoir, so answer choice (A) is
choice (E) correctly evaluates the flaw in Miteki's response: she
incorrect.
fails to address the core of Lito's comments and instead develops
a secondary argument.
B: Herbert Read comments on Renoir carrying on the "artistic
tradition that started at Rubens and ended at Watteau," but this
The Incorrect Answers: statement does not indicate Read's personal opinion about
whether or not all three men may be considered the greatest reasoning in answer choice (C) most closely parallels the
artists in Western history, so answer choice (B) cannot be principal's reasoning.
correct.
The Incorrect Answers:
C: Read notes that Renoir is the "final painter" of a certain
artistic quality, but he makes no comment about artists after A, D: Answer choices (A) and (D) are incorrect, because the
Renoir, nor does he suggest that no great artists have arisen since conclusions reached in both arguments do not parallel the
Renoir. It is, of course, possible that he makes this claim principal's conclusion in any way. The argument in choice (A) is
elsewhere in his analysis, but the claim is not recorded in this that parents should read in order to help improve their children's
particular passage, so answer choice (C) cannot be inferred and speech development skills. The argument in choice (D) is that
is thus incorrect. students should take certain kinds of high school classes to be
successful in college. Both answer choices suggest that a specific
E: Again, Read makes no comment on whether or not Rubens action should be taken to achieve results of certain quality, but
(Flemish), Watteau, and Renoir (both French) were the greatest the formula quantity = quality is not present in any form in either
painters of the seventeenth and eighteenth centuries. What is answer choice.
more, Read notes that Renoir carries on a tradition that "runs
directly from Rubens to Watteau," so it is very possible that the B: This answer choice seems promising at first, with the
unnamed painters who fell between Rubens and Watteau were conclusion that the number of stars identifying a resort (quantity)
not Flemish or French. translates automatically to the comfort of a stay there (quality).
But unlike the principal's conclusion, this conclusion does not
Question 7 require a specific change in the quantity of action or investment
to reach it, so it is not close enough to parallel it. Had answer
Overview: This question presents a statement by a school choice (B) indicated that the resorts with the most stars tend to
principal regarding how well students are doing in certain be the most successful, with the conclusion that a specific resort
subject areas. The students recently completed some testing, and should improve its business plan and seek to obtain more stars, it
the principal notes that the students scored badly in the math would be comparable. As it is, though, answer choice (B) may be
tests compared to their performance on reading tests. The eliminated.
principal then notes that students spend more class time on
reading than on math and concludes that students need to spend E: Answer choice (E) has no relevance to the principal's
as much time on math as on reading in order to improve the math conclusion and can be eliminated immediately. The conclusion
scores. The question asks the student to compare the flaw in the in (E) relates only to quantity; the question of quality does not
principal's reasoning to the flaw in the reasoning of the answer arise at any point in this particular example.
choices.
Question 8
In order to find an answer choice with a comparable flaw, it is
first necessary to identify how the principal's argument proceeds Overview: Question 8 presents a scenario in which the CEO of a
and then decide where the flaw develops. The principal first fast-food chain makes an announcement about upcoming
looks at the test results of two different subjects and sees that the revisions to the chain's menu. The company is completely
students did badly on one and well on the other. Looking for a overhauling its menu items so that all items offered will now be
quality that distinguishes these two, the principal realizes that the healthier. Specifically, the announcement notes that the company
students spend less time on math than on reading. Thus, the will be removing unhealthy fats and offering smaller portions.
conclusion follows that students should spend as much time on The question asks the student to consider which of the answer
math as on reading. The problem with this is that the argument choices most undermines the effect of the CEO's announcement.
assumes quantity = quality, with no regard for other To find the correct answer, it is necessary to focus on the details
considerations, such as the skills of the math teachers at the in the CEO's comments and on what, in particular, would call
school, the usefulness of the curriculum, and so forth. The into question the validity of the CEO's remarks.
correct answer choice will follow this pattern of quantity =
quality, with a specific outcome desired based on the investment
The Correct Answer:
of a certain quantity and a disregard for other significant
considerations.
A: Answer choice (A) states that although the fast-food chain is
indeed making the announced changes in its menu, it is also
The Correct Answer: retaining certain addictive additives that have, by implication,
long been in its food. As a result, the continued inclusion of
C: The reasoning in answer choice (C) is similarly flawed to the these additives calls into question the company's actual
principal's reasoning in that two items are compared (options commitment to offering healthy foods and raises questions about
trading and stock trading), and the outcome is assumed to be the claim that the company's food will be a healthier option than
based on the invested quantity (money in options vs. money in that offered by other fast-food chains. Therefore, choice (A)
stocks). The argument indicates that traders invest more money most seriously undermines the CEO's statements.
in stocks than they do in options and that they also tend to make
more money. No other considerations are mentioned in this
The Incorrect Answers:
comparison, such as the skill of the trader, the history of the
stock's movement, and so forth. The conclusion about outcome is
reached solely upon the basis of the quantity invested, so the
B: That the CEO is receiving a large bonus is potentially from the passage that he bases his conclusions on the practice of
suspicious, but it does not by itself undermine the CEO's claim a specific kind of spirituality or that the patients who
about providing healthier food options. In other words, nothing experienced positive results from the practice of spirituality were
about the bonus suggests that the menu will not in fact offer practicing the same form. Answer (B) cannot be correct.
healthier items than before; answer choice (B) does not address
evidence for or against the CEO's. There is no reason to be C: The head of the psychiatric association indicates that the
surprised that the company is rewarding the CEO in advance for positive aspects of spirituality were observed among the patients,
a plan that will benefit the company. Thus, answer choice (B) is but he does not comment anywhere on whether members of the
incorrect. psychiatric association practice spirituality or whether they
should. Rather, his conclusions relate directly to the patients
C: The response of the focus group calls into question the way practicing spirituality, and it cannot be assumed that his
that customers will respond to the food, but it does not call into conclusion is based on the importance of spirituality to the
question the CEO's comments about the healthy qualities of the association members. (It might be true that he also recommends
food. Thus, answer choice (C) may be eliminated immediately. the practice of spirituality to members, but this is an inference
from his comments, not an assumption on which they are based.)
D: While the merits of advertising to young children might be
arguable, the ads themselves seem to support the company's D: The head of the psychiatric association does say specifically
claim that it is offering healthier choices by encouraging children that depression, "in some cases can lead to an early death." He
to request the better menu items when they visit the fast-food does not suggest, however, that the result of depression is
chains. As a result, this answer choice seems to bring some inevitably death. Further, his conclusion that the members of the
validity to the CEO's claims of a commitment to better health, psychiatric association should encourage patients to practice
and it is incorrect. spirituality is not based on the assumption that depression will
always lead to death, but rather that it can shorten the lives of
E: The CEO makes no claim to having removed all items from some patients. Answer choice (D) is incorrect.
the old menu. Instead, he claims that the ingredients have been
improved and the menu revised. Answer choice (E) does more to Question 10
support the CEO's claims than it does to undermine it, so it is
incorrect. Overview: In question 10, the student is asked to consider which
of the answer choices most explains the disparity in price drop
Question 9 between the cost of crude oil per barrel and the cost of gasoline
at the pump. In order to determine the difference, it is necessary
Overview: In question 9, the head of a regional psychiatric to think about the specific statements made within the passage.
association makes a statement about a study that suggests the This question asks for inferences only inasmuch as those
importance of spirituality in helping patients live longer, inferences can be made directly from the information in the
healthier lives. The head of the association also comments that passage. Several of the answer choices provide distant
depression has been noted, in some cases, to shorten the lives of possibilities, but only one sufficiently explains the discrepancy
patients. The speaker concludes that members of the psychiatric in price without deviating from the details of the passage.
association should encourage patients to pursue spirituality in the Question 10 presents a scenario involving cause-and-effect, and
expectation that it will provide them with longer and healthier cause and effect are what students need to consider: what cause
lives. The question asks the student to consider on which directly explains the effect of a drop in the price of crude oil that
assumption the speaker's conclusion is based. is not reflected in prices at the gasoline pump?
E: In reaching his conclusion, the head of the psychiatric B: Answer choice (B) is the correct answer because it provides a
association does not distinguish between different forms of logical explanation for the disparity in price while not deviating
spirituality. As a result, the conclusion he reaches suggests that from the details in the passage. The cause suggested is that
all forms of spirituality are equally healthy and that all will be refineries have had to absorb the higher price of crude oil in
equally beneficial to the patients. Answer choice (E) therefore, recent months; this has had the effect of those refineries delaying
is, the most correct answer. passing on the lower price of crude oil to customers at the gas
pump.
The Incorrect Answers:
The Incorrect Answers:
A: While the head of the psychiatric association indicates that
some of the patients being treated by members of the association A, C: Although answer choices (A) and (C) provide some
have suffered from depression, he does not indicate that all of explanation for the drop in the price of crude oil per barrel, they
them have. Therefore, answer choice (A) may be eliminated, provide absolutely no explanation for the higher price at the
because the conclusion in the passage is clearly not based on this gasoline pump, leaving open the question regarding the disparity
assumption. in price. As a result, both may be eliminated immediately.
B: As the head of the psychiatric association only mentions the D: Answer choice (D) also offers some indication about what
practice of spirituality as being healthy, it cannot be assumed might affect the price of crude oil per barrel, but it fails to
address the discrepancy in price, that is, why the cost of crude oil
has decreased without a corresponding drop in prices at the points out that among the Anglo-Saxons it was not uncommon to find children
with Welsh
gasoline pump. If anything, answer choice (D) raises further
names. The great Christian poet Cdmon and Cdwalla, the king of Wessex in the
questions, since if there are alternative energy options, the price seventh
of gasoline should naturally decrease according to the laws of century, were both noteworthy and highly respected Anglo-Saxons who bore
supply and demand. Welsh names.
(30) From a purely practical perspective, it is unlikely that Anglo-Saxon parents
would bestow
E: Answer choice (E), although it reflects a potential result of Celtic names on their children if those names were closely associated with a
continued low crude oil prices, does not address the question of despised
language or a group of people deemed inferior. As a modern example, during
why gasoline prices are still high in comparison to crude oil World War I
prices. While it might be inferred that the loss of oil production people in England began changing their names to avoid sounding too Germanic.
companies entails less competition (and therefore higher prices), Even the
there is not enough information in the passage to imply such a royal family, up to that point bearing the name Saxe-Coburg-Gotha, changed the
family
conclusion or to select (E) as the best answer. (35) name to Windsor due to the long connection of that name with a specifically
English history.
Additionally, the respected Battenburg family in England, closely connected to
LSAT Practice Reading Comprehension Questions the
monarchy, felt the need to change their name to Mountbatten, as it had a less
Directions: Each passage in this section is followed by a group decidedly
German connotation.
of questions to be answered on the basis of what
is stated or implied in the passage. Some questions may have (40) Perhaps more significantly, David Crystal raises the possibility that the
more than one possible answer. However, you are to choose word cross, steeped
the bestanswer, that is, the response that most accurately and in important religious meaning for many English speakers, came from a Celtic
background.
completely answers the questions, and blacken the corresponding In Latin, the word is crux, and the Scandinavians rendered it kross. But there is,
space on your answer sheet. on the
whole, very little linguistic influence on early English religious terminology from
English language scholars generally agree that the modern English language the
developed Germanic languages or the Germanic peoples, who were decidedly pagan upon
from several sources: the Anglo-Saxon language, or Old English, spoken by the their arrival
Germanic (45) to England. On the other hand, the Irish Celts were enthusiastic and
peoples who migrated to the island of Britain in the fifth century; the Old Norse thorough in their
influences missionary efforts to England and other parts of Europe, and they rendered the
of the Vikings and the Danish kings of England in the ninth and tenth centuries; Latin crux
the French as cros in Old Irish and as croes in Welsh. It is highly possible that the English
(5) influence of the Norman invaders in the eleventh century; and the Latin word cross
influences of the and the Old Norse word kross were influenced by the Irish missionary work. It is
earlier Roman inhabitants and the Catholic Church. However, one mystery unlikely
remains. When that the mystery of the missing Celtic words will ever be solved satisfactorily,
the Anglo-Saxons arrived in Britain, there were numerous Celtic inhabitants but what little
dwelling (50) evidence remains suggests that the mystery can no longer be written off as a
alongside what remained of the Roman population. Why, then, did the Anglo- case of a
Saxons, and conquered people becoming linguistically obsolete.
thus the English, not absorb more of the Celtic languages? The English language
ultimately 1. Which of the following best states the main idea of the
(10) adopted very few Celtic words, so few in fact that scholars are at a loss to
explain the reason passage?
with any certainty. One thing is certain: the Celtic languages are in no way
related to Anglo- A. Although linguistic scholars do not know why the
Saxon, indeed developing from an entirely different family of languages, so there
is no English language has so few Celtic words, it can no
question that the Anglo-Saxons did not adopt Celtic words simply because they longer be assumed that the Anglo-Saxons avoided
already had Celtic words in the belief that the Celts were inferior.
very similar words of their own. So, what happened? Some scholars have B. The possible Celtic derivation of the word cross
suggested that the
(15) Anglo-Saxons already had enough words of their own and thus did not need suggests that the Anglo-Saxons interacted more closely
to borrow from with the Celts than was previously thought.
the Celts, even upon arriving in a new place. For instance, if the day-to-day C. New evidence suggests that the traditional belief about
elements of life the Anglo-Saxon, Old Norse, French, and Latin
in Britain were similar enough to those in the Anglo-Saxon homeland, the Anglo-
Saxons influences on the English language is erroneous and
would not feel the need to make use of foreign words to describe their new life. misleading.
This theory, D. The actions taken by the English during World War I
however, is inconsistent with evidence that the Anglo-Saxons borrowed everyday indicate strongly that their forebears eradicated Celtic
words
(20) from other languages such as Old Norse and French. Other scholars have words for similar reasons.
suggested the E. The appearance of Welsh names among significant
theory that the Anglo-Saxons chose to avoid the Celtic words because the Celts Anglo-Saxon figures indicates that of all the Celtic
were peoples, the Welsh had the greatest linguistic impact on
essentially a conquered people - an explanation that is strongly supported by the
rapid Anglo-Saxon daily life.
disappearance of Celts from south and central England and their subsequent
movement
2. The use of the word connotation in line 38 most closely
north and west into what would become Cornwall, Wales, and Scotland.
(25) suggests which of the following?
Leading linguistic scholar David Crystal disagrees with this latter hypothesis,
however. He
A. Clear relationship 7. The passage suggests that the author would probably
B. Linguistic origin agree with which one of the following?
C. Theoretical definition
D. Potential association A. There is less Latin and Old Norse influence on the
E. Emotional correlation English language that there is Celtic influence.
B. Although there seem to be few Celtic words within the
3. The discussion of the word cross in the passage is intended English language, these words suggest a significant
to show which of the following? linguistic role.
C. The possible Celtic derivation of the English word cross
A. Although they were previously ignored by scholars, it is alone suggests that the English viewed the Celts
clear that many important Celtic words were indeed favorably.
absorbed into the English language. D. The Anglo-Saxons did not adopt many Celtic words
B. Scholars now realize that many Celtic words influenced because they had enough everyday words in their own
Old Norse words and not the other way around. language.
C. It is incorrect to assume that there was a very great E. Because some of the Anglo-Saxons gave their children
influence on the English language from Celtic words. Welsh names, the Anglo-Saxon people unquestionably
D. Linguistic scholar David Crystal believes that Celtic had a high opinion of the Celts.
words make up an important part of the English
language. The question of beauty has captivated and frustrated artists, writers, and scholars
for
E. The significance of the few Celtic words within the
centuries. What defines beauty, and who can justifiably be described as
English language suggests a more important influence beautiful?
than was previously thought. Is there a universal "look" that is beautiful in contrast to one that is not? History
and literature are
replete with descriptions of women who are said to be the "most beautiful."
4. The author provides examples of English behavior toward Greek myth
German last names during World War I in order to do (5) claimed that Helen of Troy was the most beautiful woman in the world, with
which of the following? a "face that
launched a thousand ships," as well as a ten-year war between Greeks and
Trojans. But
A. Prove definitely that human nature does not change there is no clear account of Helen's appearance, and even Hollywood has been
B. Undermine the theory of the Welsh influence on unable to
agree on this issue, rendering Helen a variety of ways in different movie
English names productions about
C. Use a fairly recent event to provide context for a the Trojan War. Today, scientists are beginning to consider the question of
hypothesis beauty and to
D. Show that the English changed names because they (10) devise tests that attempt to quantify attractiveness. The goal is to see
whether beauty is
considered Germans inferior simply a subjective perception or if people in general tend to agree on who is
E. Suggest that many of the so-called "English" names are beautiful and
really German who is not. In some tests, participants from a variety of cultural backgrounds are
asked to
compare different faces and to decide who is beautiful and who is not. In the
5. Which of the following best describes the author's attitude majority of
toward the theory that there are few Celtic words in the these tests, the participants agree in large percentages that the faces most likely to
English language because the Celts migrated and had no be
(15) considered beautiful are in fact beautiful. Some scientists have even tested
contact with the Anglo-Saxons? infants, showing
them pictures of different faces. The tests indicate that the gaze of the infants
A. Self-righteous insistence tends to linger
more on the beautiful faces rather than on the faces not traditionally considered
B. Scholarly disagreement beautiful.
C. Patronizing disapproval
D. Justifiable concern Other scientists have found that this potential for a universal appreciation of
E. Vitriolic dissent beauty leads to
(20) interesting consequences. As psychologist Nancy Etcoff notes, beauty can
affect belief in
6. The primary purpose of the passage is to do which of the one's character. That is, beautiful people are often assumed to be better than
following? unattractive
people in terms of character or other traits. There is evidence that in classroom
situations
A. Caution against making an historical judgment without beautiful children are often scored higher than unattractive or less beautiful
considering further linguistic evidence children,
apparently under the assumption that the beautiful children must be doing well.
B. Introduce a new theory and support it with linguistic Scientists
evidence (25) are quick to note that this is not a conscious decision on the instructor's part
C. Defend a scholarly position by citing leading authorities but is an
in the field unconscious response to the child's appearance. Even adults are not immune to
this bias. In
D. Dispute a long-held scholarly position by disproving the applying for jobs, attractive applicants often receive the desired employment,
linguistic evidence in support of it whether or
E. Compare several theories and argue in support of one of not they are more qualified than a less attractive applicant. Political analysts also
them claim that
the attractive candidate wins the election, regardless of his or her political
platform.
(30) Historians have found that during the Nixon and Kennedy debates of 1960,
viewers tended 10. Which of the following best expresses the reasoning
to favor Kennedy in the video debates and Nixon in the radio debates, and it has
been
behind the author's main argument?
suggested that Kennedy's attractive and youthful appearance gave him the
leading edge. A. Beauty is so subjective that much more testing is
Scientists claim that a certain combination of features is universally considered
required to obtain any definitive results about what
beautiful, constitutes beauty.
(35) but one element that has not been tested to any great extent is the effect of B. Infants can identify beauty because they recognize the
personality on qualities of a person's character, as well as the features
beauty. In the immediate sense, beauty might be associated with character, with
beautiful
on a person's face.
people assumed to be morally or ethically better than others. But perhaps there is C. Given the mystery that sometimes surrounds
a reverse perceptions of beauty, it is likely that beauty is
relation to consider. That is, character might affect beauty, and not the other way determined by more than an arrangement of facial
around.
Etcoff hints at the effect of character or personality in determining beauty: she
features.
records that D. Beautiful people have received inappropriate privileges
(40) upon meeting the writer George Eliot, a woman who was generally that have created long-term consequences in society.
considered to be very E. Because scientists have been able to quantify beauty, it
unattractive, her fellow writer Henry James was immediately struck by her ugly
appearance; but in the course of talking with her for only a few minutes he
is indeed possible to determine that certain physical
discovered an features are beautiful.
inner beauty that completely altered his opinion of her outward appearance. It
may be that
although physical features are indeed important in determining beauty, beauty
11. The information that in classroom situations teachers
itself is not respond differently to attractive students than to
(45) simply "skin deep" and can be defined by more than an arrangement of eyes, unattractive students is intended to do which of the
nose, and lips. following?
Scientists studying the phenomenon of beauty would do well to turn their
attention to the
more intangible qualities that define beauty and to consider what lies beneath the A. Indicate that beauty sometimes plays a role in the
skin in perception of a person's character or qualities.
addition to what lies on it.
B. Illustrate that society gives unnecessary privileges to
the beautiful
8. Which of the following best summarizes the central idea of C. Show that the personality or character of the
the passage? schoolchildren affected their outward appearance
D. Suggest that youth is inevitably associated with beauty
A. Despite society's claim that all are beautiful in their own E. Indicate that this is a phenomenon that appears around
way, scientific studies show that beauty is quantifiable the world
and some faces are always considered beautiful.
B. Scientists testing for beauty need to consider more than 12. The example of Henry James's meeting with George Eliot
mere outward appearances to determine what makes a is intended to indicate which of the following points?
person beautiful.
C. Beauty is related far more to personality and character
A. There might be intangible qualities that contribute to the
than it is to outward appearances.
perception of beauty.
D. Beauty is inevitable, and the beautiful will always find
B. George Eliot would be considered beautiful today by
more success than those who are less attractive.
scientific standards.
E. Because beauty is subjective, scientists will never
C. It was possible to understand George Eliot's beauty only
succeed in quantifying beauty, despite developing
by interacting with her.
complex tests in an effort to do so.
D. Henry James abandoned his unnecessarily high standard
of beauty after meeting George Eliot and coming to
9. It may be inferred from the passage that the author know her better.
believes which of the following? E. Standards of beauty are universal, so if George Eliot
were considered unattractive then, she would be
A. After long ignoring beauty, scientists have finally considered unattractive now.
recognized its importance and have begun to quantify
beauty in objective tests. 13. Which of the following phrases best replaces the use of
B. There is a specific combination of features that is the word phenomenon, in describing beauty, in line 46?
considered beautiful across cultural boundaries.
C. Beauty should never be associated with a person's
A. Unexpected reality
character or personality, because personality cannot
B. Intriguing occurrence
determine outward appearance.
C. Strange quality
D. Beauty is determined entirely by cultural standards, and
D. Abstract experience
what one culture perceives as beautiful another might
E. Subjective analysis
dismiss as unattractive.
E. Personality might play a significant role in determining
whether or not a person will be perceived as beautiful. 14. The author's tone toward the claim that beautiful people
are often assumed to be of good character and receive
privileges for their appearance can best be described as
which of the following?
A. Combative skepticism A: Although the first paragraph seems to provide a great deal of
B. Mocking amusement information about the background of the English language, as
C. Quiet resignation well as theories about the lack of Celtic words, it is the final
D. Informative warning sentence of the first paragraph and the first sentence of the
E. Righteous anger second paragraph that indicate the direction the passage will be
taking. The author notes that one theory in particular seems to
Answers have support, but then goes on to say that a leading linguistic
scholar disagrees with it. The rest of the passage explains
Crystal's evidence that weakens the initial theory, and the final
Questions 1-7
sentence of the third paragraph provides the main point: "It is
unlikely that the mystery of the missing Celtic words will ever
Synopsis: This passage discusses the linguistic issue of why be solved satisfactorily, but what little evidence remains suggests
there are not more Celtic influences in the English language. The that the mystery can no longer be written off as a case of a
author begins by discussing the linguistic influences that scholars conquered people becoming linguistically obsolete." Answer
already recognize and have identified - Old English (or the choice (A) most closely summarizes this, so it is the correct
language of the Anglo-Saxons), Old Norse, French, and Latin - answer.
and then proceeds to explain that scholars consider the lack of
Celtic words to be a mystery. When the Anglo-Saxons arrived in
The Incorrect Answers:
Britain, a considerable population of Celts lived there already.
Yet there are very few Celtic words in the English language,
leading scholars to wonder why the Celts appeared to make no B: Answer choice (B) is incorrect, because it focuses on a
linguistic impact on the Anglo-Saxons. The author notes that supporting piece of evidence Crystal has provided to undermine
scholars have offered several theories. Some argue that the a theory, but that is not, in itself, the main point of the passage.
Anglo-Saxons did not need the Celtic words, because they had Answer choice (B) is incorrect.
enough of their own; a problem with this view is that the Anglo-
Saxons borrowed plenty of words from other languages. Some C: Although the author does suggest that there might be
scholars have suggested, then, that the Anglo-Saxons viewed the considerable significance to the few Celtic words that are in the
Celts as inferior people and thus avoided their language. The fact English language, the author does not claim or suggest anywhere
that the Celts ultimately migrated north and west - away from the that these words render other linguistic sources less significant.
Anglo-Saxons - seems to support this theory. Additionally, the author does not indicate that there is any
evidence contradicting the traditionally recognized influences.
In the second paragraph, the author considers this theory and Answer choice (C) is clearly incorrect.
raises questions about it, citing linguistic scholar David Crystal,
who points out that there is considerable evidence that the D, E: As in answer choice (B), the example of the English
Anglo-Saxons gave their children Welsh (or Celtic) names. For changing German names during World War I, as well as the
example, there is a record of an Anglo-Saxon king and a example of the occurrence of Welsh names among the Anglo-
renowned religious poet bearing Welsh names. It is unlikely that Saxons, is intended to give supporting evidence; such examples
the Anglo-Saxons would deliberately choose names from the are not meant to represent the primary argument. In addition,
language of a group of people believed to be inferior, so Crystal answer choice (E) states, "of all the Celtic peoples the Welsh had
points out that this theory does not bear up under scrutiny. the greatest linguistic impact on Anglo-Saxon daily life," an idea
unsupported by the passage. Answer choices (D) and (E) are
In the third paragraph, the author again cites Crystal with his incorrect.
suggestion that the English word cross might derive from Celtic
sources. Again, it is unlikely that the Anglo-Saxons would adopt Question 2
a word with such religious significance from a despised
language. The author concludes by noting that the mystery of Overview: In question 2, the student is asked to select a
why there are so few Celtic words in the English language might synonymous phrase for a word that is used in the passage. The
never be solved, but that the theory that the Anglo-Saxons student will need to consider the word itself, with its dictionary
ignored Celtic words because the Celts were viewed as inferior definitions, and then place the word within the context of the
ultimately does not have much historical or linguistic support. sentence and consider how it is being used. (If the student does
not know the meaning of the word, the student should infer what
Question 1 basic idea is intended by considering the context in which the
word appears.) In question 2, the word in question
Overview: Question 1 asks the student to consider the main idea is connotation, which is defined as a connection, association, or
of the passage. This question does not ask for any inferences, so secondary meaning. In the passage, the word is being used to
the student just needs to consider the author's argument and convey the English concern that their names were connected to
summarize it. Each paragraph contains a topic, and these topics German names or assumed to be German. With this in mind, the
ultimately contribute to a primary point. What the student needs student needs to consider the answer choices.
to watch for in the answer choices are options that mention a
supporting idea but that do not reflect the main point of the The Correct Answer:
passage. The correct answer choice must encompass the
evidence in the passage with a single statement. D: Of all the answer choices, answer choice (D) best conveys the
idea of "connection" and "assumption" with the phrase potential
The Correct Answer: association. As the passage notes, the English changed their
names "to avoid sounding too Germanic." In other words, they A: The passage does not claim that many Celtic words
feared the potential for their names being associated with influenced English; rather, the passage notes that there are few
German names. Answer choice (D), therefore, is correct. such Celtic words. Answer choice (A) is incorrect.
The Incorrect Answers: B: The discussion of the word cross does suggest that an Old
Irish (Celtic) word might have influenced an Old Norse word,
A: Answer choice (A) offers a good option, but the passage does but the passage does not indicate at any point that there were
not suggest that the English knew there would be a clear other Old Irish words that influenced Old Norse words. Answer
relationship, nor does the word connotation in the context of the choice (B) makes assumptions that are not supported by the
passage suggest a clear relationship. Instead, it suggests the passage, so it can be eliminated immediately.
possibility of a connection between English names and German
names, a connection that answer choice (A) does not indicate. C: The point of the discussion of the word cross cannot be to
Answer choice (A) is incorrect. illustrate that it would be wrong to assume that the Celtic
language had a great influence on English. The author is not at
B: Although the fears of the English were about a linguistic pains to illustrate that one should not do so; rather, the author
matter (they were concerned that their names might be connected assumes that no one will make the assumption that the Celtic
with German names), the phraselinguistic origin cannot replace language had a very great influence on English. Answer choice
the word connotation in meaning. Answer choice (B) is (C) is incorrect.
incorrect.
D: David Crystal's contribution to the passage is to show that
C: Although the word connotation can, in some cases, indicate a there are signs of Celtic influence on the English language. The
definition that is more theoretical than it is concrete, there is passage does not make any claim, however, that Crystal believes
nothing in the use of the wordconnotation in the passage to that Celtic words make up an important part of the English
suggest that the English were concerned about atheoretical language. Answer choice (D), therefore, is incorrect.
definition. Answer choice (C) is incorrect.
Question 4
E: Although the decision by some English people to change their
names might have been based on emotion, the passage does not Overview: As in question 3, question 4 asks the student to
suggest this. Because there is no clear suggestion of emotion in consider the role of the example about English actions toward
the passage, answer choice (E) is incorrect. German names during World War I. And as with the discussion
of the word cross, this particular information offers secondary
Question 3 details that support the main point of the passage. The student
needs to consider the English/German names discussion, then,
within the context of the main point, as well as within the
Overview: Question 3 asks the student to consider the purpose of
the discussion ofcross in the third paragraph. It was noted in the context of the statements immediately around it. The correct
explanation for question 1 that the possible origin of the answer might or might not mention specifically the main point of
the passage, but it will show that the example fits well with that
word cross represents supporting evidence for the main point of
main point.
the passage. Therefore, the student should immediately
recognize that the correct answer will indicate the way in which
this discussion supports or points to the main argument. Recall The Correct Answer:
that the main argument of the passage is that, despite the rarity of
Celtic words in English, linguistic evidence suggests an C: The discussion of the English decision to change German or
influence on the English language from some Celtic words, so it German-sounding names during World War I follows a
is no longer possible to claim that the Anglo-Saxons deliberately paraphrased reference from David Crystal about the fact that the
avoided the people or the language in the belief that the Celts Anglo-Saxons would probably not have given their children
were inferior. Welsh names if those names were associated with something or
someone negative. Since there is no immediate connection to
The Correct Answer: Welsh names and German names, the student can assume that
the author is intending to use this particular example to show
how perceptions of peoples (whether they are Germans or Celts)
E: Bearing the main point of the passage in mind, answer choice
(E) is the only answer choice to place the cross discussion within influence the use of names associated with those peoples.
Answer choice (C) expresses this idea and is thus the correct
the context of this main point. Choice (E) notes that the
answer.
information about the word cross offers evidence that the few
Celtic words that exist in the English language are quite
significant; this indicates that the Celtic influence might be more The Incorrect Answers:
important than was previously thought. This point supports the
main argument, so answer choice (E) is the correct answer. A: The passage makes no reference to human nature, and
although it might be thought that associating names from certain
The Incorrect Answers: linguistic backgrounds with either positive or negative qualities
is a facet of human nature, there is not enough information in the
passage to support this point. More specifically, the question of
human nature is not related to the larger question of the impact
of certain Celtic words in the English language. Therefore, and this passage does not suggest a patronizing tone at any point.
answer choice (A) cannot be correct. Had the author mentioned the differing viewpoint repeatedly in
order to belittle those who held it, the tone might be described as
B: If the discussion of the English response to German names patronizing. As it is, however, the author mentions the viewpoint
were being used to support the traditional theory about Anglo- only twice and the scholars holding the view once, allowing the
Saxons viewing the Celts as inferior, this argument might indeed rest of the discussion to focus on evidence that supports his own
undermine the theory of Welsh influence. But in the context of perspective.
the second paragraph, it actually supports the point about Anglo-
Saxons embracing certain Welsh influences, because it indicates D: The author's concern about the traditional viewpoint might be
clearly that the Anglo-Saxons did not avoid Welsh names. justifiable in his own mind, but the passage does not necessarily
Answer choice (B) is incorrect. convey this tone. Instead, the tone is a scholarly one that leaves
emotion at the door and relies on evidence. Answer choice (D) is
D: Answer choice (D) seems briefly promising, because the incorrect.
example of the English immediately follows this statement by
the author: "it is unlikely that Anglo-Saxon parents would E: Although the author does disagree with the traditional
bestow Celtic names on their children if those names were viewpoint, there is nothing in the passage to indicate vitriol.
closely associated with a despised language or a group of people Answer choice (E) can be eliminated immediately.
deemed inferior." This seems to suggest that if the English
during World War I did to the Germans what the much earlier Question 6
Anglo-Saxons did not do to the Welsh, then the English might
have viewed the Germans as inferior. But once again, this is not Overview: In question 6, the student is asked to consider the
the main point of the passage. Answer choice (D) is thus primary purpose of the passage. This is slightly different from
incorrect.
considering the main point of the passage in that the student is
looking from an even broader perspective without necessarily
E: The passage does not indicate at any point that English names paraphrasing and summarizing details. However, knowing the
might really be German. In fact, it seems to suggest just the main point is helpful in that the main point shapes the primary
opposite - that if many of the English changed German names, purpose of the passage. In this case, the author's final statement
the names are now English. More to the point, however, this provides some indication of this: "what little evidence remains
inference does not have a strong connection to the main point of suggests that the mystery can no longer be written off as a case
the passage, and therefore answer choice (E) is incorrect. of a conquered people becoming linguistically obsolete."
Clearly, the author is suggesting that there is evidence to counter
Question 5 a specific viewpoint while discouraging embracing this
viewpoint without considering the evidence.
Overview: Question 5 asks the student to consider the author's
tone toward the traditional argument that the Anglo-Saxons The Correct Answer:
might have deliberately avoided Celtic words because they
viewed the Celts as inferior. Certain key words in the passage A: Answer choice (A) reflects the intention of the passage, as
will help discern whether the author's tone is one of vitriolic displayed in the final sentence: the author's purpose is to caution
disagreement, patronization, or something else altogether. Such against making an historical judgment (the traditional viewpoint
phrases include "Other scholars have suggested the theory...," about the Anglo-Saxons deliberately avoiding Celtic words)
"leading linguistic scholar David Crystal disagrees...," "the without considering the further linguistic evidence (the Welsh
mystery can no longer be written off..." All of these turns of names and the wordcross). Answer choice (A) is thus correct.
phrase suggest a polite scholarly discussion in which one scholar
(the author) disagrees with other scholars - firmly but not
The Incorrect Answers:
necessarily rudely. The student should select an answer choice
that best reflects this.
B: The author does not necessarily indicate that the theory
advocated is a new one; in fact, the author suggests that the
The Correct Answer:
leading authority David Crystal holds this theory, so the author is
doing more to summarize Crystal's theory than to produce a new
B: Answer choice (B) is the only answer choice to present the one. Answer choice (B) does not have enough support within the
best description of the author's tone: scholarly disagreement. The passage, so it is incorrect.
author is polite but holds to a certain view and defends that view.
Answer choice (B), therefore, is correct.
C: The author is indeed discussing Crystal's opinion, but the
purpose of the passage is not so much to defend his position but
The Incorrect Answers: rather to caution against holding the traditional theory with
respect to the linguistic evidence. Answer choice (C) is incorrect.
A: The author might hold firmly to an opinion, but there is no
tone of insistence, nor is the author self-righteous at any point. D: The author does attempt to disprove a traditional theory, but
Answer choice (A) is clearly incorrect. there is no real focus on the linguistic evidence that supports this
theory. (In fact, the author does not even mention the linguistic
C: The author clearly disagrees with the traditional viewpoint, evidence that supports it and mentions only historical evidence
but disagreement alone does not guarantee a patronizing attitude; of the relocation of the Celts.) Answer choice (D) is incorrect.
E: The author discusses two of the opposing theories, but there is Synopsis: Questions 8-14 are based on a reading passage that
no comparison in the passage. Answer choice (E) cannot be discusses beauty and whether it can be objectively quantified.
correct. The author begins by presenting the general point that beauty has
always fascinated people and then mentions that in the modern
Question 7 era scientists have begun considering beauty from a forensic
perspective, asking whether beauty can be measured and
quantified scientifically. The author writes that scientists believe
Overview: The final question for this Reading Comprehension
that they can measure and quantify beauty, and their tests - based
passage asks with which answer choice the author would likely
on the results from volunteers of many social and cultural
agree. As with all questions like this, the student needs to
backgrounds - indicate that there are some people who are
consider what is stated directly in the passage and what can be
inferred from these statements. The student should also take such definitely, consistently perceived as beautiful and some who are
qualities as tone into account. not. The author states that even infants are not immune to beauty
and that studies indicate that babies are drawn to beautiful faces.
The Correct Answer:
The author begins the second paragraph with a hint of caution,
however. Although beauty might be quantifiable and objective,
B: The author notes in the first paragraph that there are few the response to beauty has potentially serious consequences. The
Celtic words in the English language and then implies in the author cites the research of psychologist Nancy Etcoff to show
second and third paragraphs that althoughthere are few words, that beauty is often confused with goodness and that beautiful
these words suggest a significant linguistic role (significant people are, in many cases, assumed to be good in some sense.
because the word cross itself, for example, has religious Teachers tend to score attractive students higher; employers tend
significance). Answer choice (B), therefore, is correct. to award jobs to the attractive applicants; even voters tend to
place their support behind the attractive candidate. The author's
The Incorrect Answers: tone suggests that this is a concern worth looking into more
closely.
A: Although the author suggests that one word in the English
language might have a Celtic origin as opposed to an Old Norse In the third paragraph, the author discusses the question of
origin, it cannot be inferred that the author believes there are whether people are looking at the beauty/character relation the
more words (in English) of Celtic origin than of Latin or Old wrong way around. Perhaps beauty and character (or
Norse origin. In fact, the author states several times that there are personality) are related, but it is character or personality that
not many Celtic words in the English language, so the passage affects beauty. Once again, the author cites Etcoff, this time with
does not support the inference that one word is indicative of a an anecdote about the notoriously unattractive writer George
much broader trend. Answer choice (A) is incorrect. Eliot. Although author Henry James at first thought her ugly, he
revised this view significantly after he had spoken with her and
C: The author uses the example of the word cross to indicate the come to know her better. The author concludes by suggesting
potential for significant Celtic influences on English. Although that scientists look more closely at the way that character affects
this is related to the issue of how the English viewed the Celts beauty, or the perception of it, because character might play a
(favorably or unfavorably), the passage does not indicate that the much larger role in beauty than previously thought.
author believes that the origin of the word cross alone signifies
how the English viewed the Celts. The author also cites David Question 8
Crystal's mention of the Welsh names; so, it cannot be inferred
that the author believes that cross alone is significant in regard to Overview: Question 8 asks the student to summarize the main
the issue of how the English viewed the Celts. Answer choice point of the passage by considering the author's central idea. The
(C) is incorrect. student should recognize that the primary point of the passage
lies in the last part of the final paragraph, when the author
D: The author cites the information in answer choice (D) as one recommends studying how character or other such personal
reason why some scholars believe that Anglo-Saxons did not qualities affect beauty: "Scientists studying the phenomenon of
absorb many Celtic words. However, the author also points out beauty would do well to turn their attention to the more
that this theory "is inconsistent with evidence that the Anglo- intangible qualities that define beauty and to consider what lies
Saxons borrowed everyday words from other languages such as beneath the skin in addition to what lies on it." The answer
Old Norse and French." Answer choice (D) must be incorrect. choice that best paraphrases this sentence will be the correct
answer.
E: The author notes that it is unlikely that the Anglo-Saxons
would have given their children Welsh names if they believed The Correct Answer:
the Celts to be inferior, but it cannot necessarily be inferred from
this alone that the Anglo-Saxons unquestionably had a high B: Answer choice (B) best summarizes the final sentence of the
opinion of all Celts. This strong claim goes well beyond third paragraph by stating that scientists need to consider
anything stated or implied in the passage. Answer choice (E) is qualities other than those on the face in order to study beauty as
incorrect. accurately as possible.
D: The author notes that scientists believe some people will Question 10
always be perceived as beautiful, observing that beautiful people
often receive privileges on the basis of their outward appearance. Overview: Question 10 asks the student to determine which
But this is not the author's main argument, so answer choice (D)
answer choice best expresses the reasoning behind the author's
is incorrect.
argument. This question essentially requires the student to
identify the main argument and then identify the reasons the
E: At no point does the author indicate that scientists will be author gives for that argument. All information necessary for
unable to quantify beauty; in fact, the author suggests that identifying the correct answer is contained within the passage.
scientists have already been fairly successful in quantifying The main point is that scientists, in their studies of beauty,
beauty (lines 16-24), so answer choice (E) contradicts statements should consider studying character or personality alongside their
that the author makes in the passage. Answer choice (E) is study of physical beauty. This suggests that the author's
incorrect. reasoning is that there must be something more than outward
appearance that determines beauty. The answer choice that best
Question 9 expresses this idea will be correct.
Overview: Question 9 asks the student to select an answer choice The Correct Answer:
that, on the basis of the passage, infers the author's belief. The
student should keep the passage closely in mind and use only the C: Answer choice (C) best summarizes the reasoning that inner
direct statements of the passage to determine the correct answer. qualities might affect outward appearance. This answer choice
In many cases, wrong answer choices will contain bits of includes the author's statement from the first paragraph that
information thatseem correct. But the question is not asking what beauty often seems mysterious, as well as alluding to the
the author seems to believe; the question is asking the student to statements from the last paragraph about Henry James perceiving
infer a correct answer from what the author doessay. George Eliot as more beautiful after he had come to know her
better. Answer choice (C) is correct.
The Correct Answer:
The Incorrect Answers:
E: In the third paragraph, the author makes the following
statements: "Character might affect beauty, and not the other A: Although the author does encourage scientists to expand their
way around," and "It may be that although physical features are approach to studying beauty, the author does not mention the
indeed important in determining beauty, beauty itself is not idea of "definitive results" anywhere in the passage, nor is the
simply skin deep' and can be defined by more than an author's focus on whether or not such results can be achieved.
arrangement of eyes, nose, and lips." From this, the student may Answer choice (A) is incorrect.
deduce that the author is suggesting that personality might affect
the perception of beauty. Answer choice (E) correctly
B: Despite the fact that the author mentions the scientific study
summarizes the author's implied belief and is thus correct.
about infants recognizing beauty, there is no indication in the
passage that this is due to infants also recognizing character.
The Incorrect Answers: What is more, this answer choice does not express the reasoning
behind the author's main point, so it cannot be the correct
A: The author does note that, "Today, scientists are beginning to answer. Answer choice (B) is incorrect.
consider the question of beauty." However, this does not in itself
imply that scientists have long ignored beauty (this is a stronger D: Although the author does not seem to support beautiful
claim than what the author suggests) but only that today's people receiving privileges just on the basis of their beauty, the
scientific community has begun studying beauty. Answer choice author makes no mention of long-term consequences in society
(A) is incorrect. (mentioning only "interesting consequences"), so answer choice
(D) has no support in the passage. In addition, answer choice (D)
B: Although the author opens the third paragraph with the does not describe the reasoning behind the author's main point,
statement, "Scientists claim that a certain combination of so it can be eliminated immediately.
features is universally considered beautiful," the author does not
E: Although the author does indicate the reasoning described in does the author indicate that the studies he cites (regarding how
answer choice (E), this answer choice does not clearly express teachers respond to attractive students) can be described as
the reasoning behind the author's mainpoint, relating instead to a universal. Answer choice (E) infers more than the passage
secondary point. Answer choice (E), therefore, is incorrect. supports, so it is clearly incorrect.
Question 11 Question 12
Overview: Question 11 asks the student to consider the Overview: Question 12 asks the student to review the
discussion of how teachers respond to attractive students in information about Henry James's meeting with George Eliot and
classroom situations and then place this discussion within the to identify the purpose of this particular discussion, that is, what
context of the passage; in particular, the student is to identify the the author is trying to indicate by including it. To select the
purpose of this information in the passage. First the student correct answer, the student needs to place the Henry
should place the discussion within the context of the main point James/George Eliot discussion within the context of the main
and then examine it in relation to the surrounding sentences. The point as well as the surrounding sentences. The main point, of
main point states that scientists should consider character along course, is that scientists should consider studying character or
with physical features in studying beauty. The sentence personality when studying beauty. In the sentence immediately
immediately before the beginning of the teacher/student before the anecdote about James and Eliot, the author notes,
discussion is, "Beautiful people are often assumed to be better "Etcoff hints at the effect of character or personality in
than unattractive people in terms of character or other traits." determining beauty." From this, the student can determine that
Therefore, the correct answer will reflect the idea that attractive the purpose of this particular section in the passage is to indicate
people are often assumed to have better character (or other that beauty is not always determined exclusively by outward
superior traits) than unattractive people. appearance and that qualities of personality or character can be a
significant part of the perception of beauty. The answer choice
The Correct Answer: that best expresses this will be correct.
A: Answer choice (A) best reflects the idea that character is The Correct Answer:
often associated with appearance. Additionally, answer choice
(A) is suggestive of the information that teachers scored A: Answer choice (A) is the only answer choice that effectively
attractive students higher, thereby perceiving their qualities on conveys the idea that personality or character can affect the
the basis of their looks. Answer choice (A), therefore, is correct. perception of beauty. Thus it is the correct answer.
B: Although the author does not indicate approval of the idea B: Although the author does say that George Eliot was "a
that beauty can determine character in the eyes of some, the woman who was generally considered to be very unattractive"
author also does not explicitly discuss disapproval or express the (indicating that this perception was largely exclusive to her day,
view that beautiful people receive "unnecessary privileges." The since the modern person would be unable to offer as precise an
information about teachers and attractive students in classroom opinion), the author does not make any suggestion that George
situations is not intended to illustrate that there is a problem with Eliot would be judged differently today, even by the "scientific"
unnecessary privileges; rather, it functions as evidence for the tests referred to in the passage. In fact, the author notes that
statement that perceptions of beauty influence perceptions of scientific studies show that perceptions of beauty based on
character. Answer choice (B) is incorrect. outward appearance alone are universal, so there is no reason to
think that Eliot would be judged differently today. Answer
C: The author does not indicate anything about the character of choice (B) is clearly incorrect.
the students except to say that it was perceived to be good based
on the how the students appeared. This is clearly not the same as C: Although the author does tell the anecdote to indicate that
the claim that character affects how people appear (rather, it is Eliot's personality or character played a significant role in
the reverse). Answer choice (C) is incorrect. James's perception of her beauty, there is no clear suggestion that
the only way to perceive her as beautiful was by interacting with
D: There is no discussion in the passage about youth affecting her. Answer choice (C) makes inferences that are unsupported by
beauty (in the discussion of the school children, there is no claim the passage, so it is incorrect.
that children are perceived as beautiful because they are young;
in fact, the discussion explicitly mentions that some children are D: The author makes no comment on James's standards of
seen as less attractive, rather than as uniformly beautiful). The beauty. In fact, the placement of the story within the passage
author does cite research showing that employers hire applicants suggests that James is intended to represent the average person
based in part on the beauty of applicants and that voters choose with a standard appreciation for certain physical features, so it is
candidates based in part on the beauty of candidates. But there is inaccurate to claim from the passage alone that James's standards
no support in the passage for the idea that youth is associated were different than the standards of others. Answer choice (D) is
with beauty. incorrect.
E: Although the author does mention that there are some features E: Although answer choice (E) is essentially correct in its
universally deemed beautiful, there is no mention of the way statement, this statement does not, in and of itself, contribute to
teachers respond to beauty in students around the world. Nor the author's reason for including the anecdote. The George
Eliot/Henry James story contributes to the author's main point, as anger or combativeness. The correct answer choice will reflect
well as supports the statements immediately surrounding it; the author's intention to provide information with a gentle hint of
because answer choice (E) does not explain how the story does warning.
this, it is incorrect.
The Correct Answer:
Question 13
D: The phrase informative warning best expresses the author's
Overview: In question 13, the student must consider the meaning tone, because it conveys the attitude that the author takes: one of
of the wordphenomenon within the context of the passage and providing information and suggesting a warning but not one of
select a synonymous phrase that best replaces it. As always with insistence or anger. Answer choice (D), therefore, is correct.
word replacement questions, the student must consider not only
the word itself but the way that it is being used in the passage. It The Incorrect Answers:
is entirely possible that several of the answer choices could be
correct, but only one of them will be correct. In the passage, the
A: The author's tone is not one of combativeness; on this basis
word phenomenon as a description of beauty is being used to alone, answer choice (A) can be rejected. In addition, rather than
suggest something that is not so much astonishing or amazing sounding skeptical, the author indicates that the information
but that does occur and is worthy of being studied. The correct
cited about the issue is accurate. Answer choice (A), therefore,
answer choice will reflect this.
cannot be correct.
The Correct Answer: B: The author does not mock the issue at any point, nor is there a
tone of amusement, since the author takes the issue seriously.
B: The phrase intriguing occurrence best replaces the Answer choice (B) is clearly incorrect and can be eliminated
word phenomenon as it is being used in the sentence, because it immediately.
indicates something that does happen (occurrence) and that is
worthy of study (intriguing). Answer choice (B), therefore, is C: The author does not directly encourage action. Nor does the
correct.
author seem to give in and assume the problem is inevitable, so
there is no sense of quiet resignation. Answer choice (C) cannot
The Incorrect Answers: be correct.
A: The passage does indicate that beauty is a reality, but it does E: The author might very well be angry, but this tone is not
not suggest anywhere that this reality is unexpected. Answer apparent in the passage. And although the author's concern might
choice (A) has no support within the passage, so it is incorrect. be righteous, this does not suggest in itself that the author's tone
is one of righteous anger. Answer choice (E), therefore, cannot
C: The author does note that beauty is often mysterious, but this be correct.
is not the same thing as strange. There is no indication in the
passage that the author considers beauty strange. Answer choice
(C), therefore, is incorrect.
E: The author suggests in the passage that beauty can be studied Questions 1-4 refer to the following passage:
scientifically, which aims at objectivity instead of subjectivity,
so the phrase subjective analysis cannot be a correct replacement When the goalie has been chosen, the Smalltown Bluebirds
for phenomenon. What is more, subjective analysis does not hockey team has a starting lineup that is selected from two
make sense as a replacement for phenomenon in the sentence groups:
("scientists studying the subjective analysis of beauty"), so it can
be eliminated immediately. First Group: John, Dexter, Bart, Erwin
Overview: The last question of the second Reading When deciding on the players in the lineup, the coach considers the
Comprehension passage asks the student to consider the author's following requirements:
tone toward the issue of beautiful people receiving privileges or
being judged as better on the basis of their beauty. The student Two players are always chosen from the first group, while three are
should note that although the author indicates disapproval of this, chosen from the second group.
there is no clear statement of anger toward it. Rather, the author's
tone is largely one of giving information. Using an expression George will only start if Bart also starts.
such as "interesting consequences," the author is not demanding
that the reader believe one thing over another, or suggesting
Dexter and Bart will not start together.
If George starts, Marlene won't start. 5. What is the name of the Dragon?
The four fastest players are: John, Bart, George and Patricia A. Audrey
B. Hamish
Three of the four fastest players will always be chosen. C. Melville
D. Rex
1. If George is in the starting lineup, who must also start? E. Dave
Melville isn't the puppet who is operated by Sue and her 9. What table is Stephen playing at, and what is the score at
assistant Pam. that table?
Hamish's chief puppeteer (who is not Jill) is assisted by Tom.
Ben is in charge of the dragon, but Jill doesn't have anything to A. Table 1, 2.5:1.5
do with the kangaroo. B. Table 1, 3:0
Dave is the assistant puppeteer for the tiger. C. Table 2, 3:0
Rex, whose chief puppeteer is Paul, isn't the gorilla (whose name D. Table 2, 2.5:1.5
is not Melville). E. Table 3, 2:1
10. Whose score is highest? Sulfur dioxide can't be processed on Fridays.
Methane can't be processed on Wednesday.
A. Mike
B. Stephen 14. What are the three chemicals that can be processed on
C. Richie any given Monday?
D. David
E. Lenny A. Liquid hydrogen, xenon, and oxygen
B. Methane, oxygen, and sulfur dioxide
11. Which player has black pieces and is tied? C. Methane, xenon, and oxygen
D. Sulfur dioxide, methane, and liquid hydrogen
A. Mike E. Xenon, oxygen, and sulfur dioxide
B. David
C. Richie 15. Which weekday is most likely to be impossible for three
D. Don chemicals to be processed in one day?
E. Terry
A. Monday
12. Who is the winning player at table 4? B. Tuesday
C. Wednesday
A. Don D. Thursday
B. Terry E. Friday
C. David
D. Gerry 16. There are three chemicals that can be processed on a
E. Richie Friday. What are they?
Question 13 refers to the following passage: A. Oxygen, methane, and liquid hydrogen
B. Liquid hydrogen, methane, and sulfur dioxide
Larry has purchased a device that the manufacturer claims will C. Liquid hydrogen, methane, and xenon
reduce the fuel consumption in his car. After a month has D. Methane, oxygen, and xenon
passed, Larry determines that his mileage currently rests at 17 E. Liquid hydrogen, oxygen, and sulfur dioxide
miles per gallon. Larry's best friend, Steve, owns the exact same
make and model car, and has calculated his mileage at 23 miles 17. On which days of the week can they process liquid
per gallon. Steve's car does not have the device that Larry hydrogen and only an additional two chemicals?
purchased one month ago. Larry then makes the conclusion that
the manufacturer of the device's claim is not true. A. Monday and Tuesday
B. Tuesday and Thursday
13. Which of the following statements would cause Larry's C. Wednesday and Friday
conclusion to be the weakest? D. Thursday and Friday
E. Monday and Friday
A. Though Larry has the same make and model of car as Steve,
Larry's car is 15 years older. Question 18 refers to the following passage:
B. Larry was driving in the city, whereas Steve drives the
highway. In a lab, the scientists have been performing tests on pregnant
C. Larry purchases a lower grade of gasoline than Steve. lab rats with caffeine. These rats were given the equivalent
D. Steve lied, he actually only gets 15 miles per gallon. amount of caffeine that a person would consume with six cups of
E. Before buying the device, Larry had never before calculated coffee every day. The caffeine increase also increased the
the mileage of his car. occurrence of birth defects. A media relations person told
reporters that the government would not require warning labels
Questions 14-17 refer to the following passage: on products that contain caffeine, as the testing was continuing
and it may have different results in the future, and that the
A petrochemical plant manufactures a range of hazardous government did not want to lose credibility.
chemical products and must therefore follow strict guidelines
concerning how each of the chemicals may interact with one 18. Of the following statements, which is the most in line with
another on a daily basis. The plant processes five different the statement that was made by the media relations person?
chemicals every week. Three of these chemicals can be
processed on any given day. A. A warning applying only to a small minority of people is
inappropriate.
Xenon may be processed any day except for every other Monday B. Six cups of coffee per day is much higher than what a person
and every other Thursday. typically drinks.
Oxygen can be processed only on Tuesdays and Wednesdays. C. The conclusive nature of studies that have been performed on
Liquid hydrogen may be processed on Mondays, Wednesdays animals is doubted.
and Fridays. D. Studies on rats don't provide us with much data regarding
birth defects in humans.
E. The significance of birth defects due to the use of caffeine is John, Bart, George, and Patricia-we know Bart and George are
unclear. starting; three of the four fastest are always chosen, leaving
either John or Patricia, eliminating Erwin.
Questions 19-21 refer to the following passage:
3. C: Marlene is starting and not among the four fastest, of
At the local butcher's shop, there were five customers in the whom three are always chosen; John and Bart-not just John (A)-
lineup. Each of the customers bought something different. are, leaving one more. Dexter is not one of the fastest, so it
cannot be John and Dexter (B). Dexter and Bart never start
The first names of the customers were Annie, Jessica, Lily, Maggie, together (D). John, Erwin, and Bart (E) are all three in the first
and Naomi. Their last names were Bore, Hazlitt, Piggott, Sowter, group, from which only two are chosen. Of the fastest four, the
and Trotter. The available products were Cumberland sausage, choices here include only John and Bart.
pork chops, pork pie, scotch eggs, and sliced ham.
4. E: Bart is the only one who must start in order for George (C)
Lily Piggott was served later than the customer who requested to start. If Bart does not start, then neither does George; but they
the sliced ham, but before Mrs. Sowter. are two of the four fastest players, of whom three are always
The second customer was Maggie. chosen. If two of the four fastest didn't start, that would leave
The pork pie was purchased by the customer directly after only two of the four fastest, not three.
Jessica.
Naomi was the woman who bought the scotch eggs; she was 5. B: Ben operates the Dragon; Paul operates Rex. Pam (NOT
served after Annie. Dave) assists Sue, so they do not operate the Tiger, OR Melville;
The Cumberland sausage was requested by Mrs. Trotter. therefore, they operate the Gorilla, and Rex is the Kangaroo. Jill
Mrs. Hazlitt was the third in line. does NOT operate Hamish, so the Tiger she and Dave operate
The fourth customer in the line bought the pork chops. must be Melville. Therefore, Sue and Pam's Gorilla is Audrey;
and Tom assists Ben with Hamish, the Dragon.
19. What was purchased by the third person in line?
6. A: Dave is the assistant with the Tiger. Pam assists Sue with
A. Cumberland Sausage Audrey the Gorilla. By process of elimination, Gale assists Paul
B. Pork pie with Rex the Kangaroo, and Tom assists Ben with Hamish the
C. Sliced ham Dragon, leaving Jill, who is assisted by Dave with the Tiger,
D. Pork chops whose name-the only name left-is Melville.
E. Scotch eggs
7. A: Tom assists with Hamish's chief puppeteer, who is not Jill
20. What was the last name of the person who purchased the (B). Paul (C) is chief puppeteer for the puppet Rex,* not Hamish.
pork pie? Sue (D) is chief puppeteer working with assistant Pam, not Tom.
*Rex (E) is not a chief puppeteer at all, but rather a Kangaroo
puppet.
A. Trotter
B. Bore
C. Hazlitt 8. C: Gale works to assist chief puppeteer, Paul, with the
D. Piggott Kangaroo puppet named Rex. The Dragon (A) puppet named
E. Sowter Hamish is operated by Ben and his assistant Tom. The Gorilla
(B) puppet named Audrey is operated by Sue and her assistant
Pam. The Tiger (D) puppet named Melville is operated by Jill
21. What place was Naomi in line?
and her assistant Dave. The passage does NOT include a lemur
(E) puppet at all.
A. First
B. Second
9. C: Stephen has lost all his games until now, so the score must
C. Third
be 3:0 in favor of his opponent. This narrows the choices down
D. Fourth
to (B) or (C). At least one game at table 1 has resulted in a tie,
E. Fifth
but Stephen has lost all of his games, so he must be at table 2
(C).
Answers and Explanations
10. D: Assuming the players and scores are each listed
1. D: John or Patricia. Of the four fastest-John, Bart, George, and consecutively by table, the leading player at table 1 is David,
Patricia-three are always chosen to start. If George is one as with the highest score of 3.
specified, that leaves John, Bart, and Patricia. Knowing George
will only start if Bart also starts, if George is in the starting
11. A: David (B) has white pieces, not black. Richie (C) has
lineup, then Bart must also be. The other two fastest are John and
black pieces, but has NOT been in a tied game. Don (D) is
Patricia; therefore, one of them is the third of the three out of the
currently leading his match and so is not tied. Terry (E) has
four fastest always chosen.
white pieces, not black. By process of elimination, the only
choice left is Mike, who has black pieces (A).
2. A: Erwin and Dexter. Bart must be starting because we know
George is starting and will only start with Bart. Dexter and Bart
will not start together, eliminating Dexter. Of the four fastest-
12. A: Currently, after three games with the fourth game in was fourth and bought pork chops. Naomi Sowter was fifth and
progress not scored yet, Don is leading his match and Richie (E) bought scotch eggs.
is not in the lead. Terry (B) is identified as playing at table 4, but
no further information is given. No information is given about 20. B: The pork pie was purchased by Maggie Bore, the second
David (C) or Gerry (D) regarding who is winning at table 4. in line following the first customer, Jessica Trotter. Following
them were (3) Annie Hazlitt, buying sliced ham; (4) Lily Piggott,
13. E: To determine whether the device increased/decreased/did buying pork chops; and (5) Naomi Sowter, buying scotch eggs.
not change his car's mileage, Larry would need a baseline By arranging the given information in a table by first name,
measurement of his car's average mileage before installing the surname, place in line, and food, one can fill in the missing
device. Without this baseline to compare with post-installation pieces through elimination and matching.
mileage, Larry's conclusion is unsubstantiated. Car ages (A), city
vs. highway driving (B), gasoline grades (C), and Steve's lying 21. E: Naomi Sowter was fifth in line, buying scotch eggs after
(D) would all affect mileages, but are immaterial when Larry Lily Piggott, who was fourth, buying pork chops and followed
cannot compare his car's current mileage to its mileage before Annie Hazlitt, who was third and bought sliced ham. Maggie
device installation. Bore was second in line, preceding Annie Hazlitt and buying
pork pie after Jessica, who was first in line and bought
14. D: Xenon cannot be processed every other Monday. Oxygen Cumberland sausage. Arranging known first names, surnames,
is only processed on Tuesdays and Wednesdays, eliminating places in line, and purchases into cells/boxes in a table/chart
Mondays. The remaining three may be processed on any given makes it easier to match/eliminate to induce/deduce missing
Monday: Sulfur dioxide on any days except Fridays, methane on pieces.
any days except Wednesdays, and liquid hydrogen on Mondays,
Wednesdays, and Fridays. Analytical Reasoning Questions
15. D: Thursdays are most difficult, because Xenon cannot be 1. There has been a sharp increase in the subscription prices of
processed every other Thursday, and oxygen (processed only on many accounting school text books in the past five years. Many
Tuesdays and Wednesdays) and liquid hydrogen (processed only publishers ascribe the necessity for these increases to the easy
on Mondays, Wednesdays, and Fridays) both cannot be availability of electronic books, which enable people to
processed on Thursdays. Mondays (A) are only restricted against electronically copy the books they want rather than buying the
processing xenon (alternate Mondays) and oxygen. Tuesdays (B) printed text.
are only restricted against processing liquid hydrogen.
Wednesdays (C) are only restricted against processing methane.
Fridays (E) are only restricted against processing sulfur dioxide. Which of the following, if true, would make this explanation
more plausible?
16. C: Liquid hydrogen is processed Monday, Wednesday, and
Friday. Methane is processed any day except Wednesday. Xenon A. The great majority of student texts have a massive backlog
is not processed on alternate Mondays and Thursdays, but on all awaiting publication.
other days including Friday. Oxygen [(A), (D), (E)] is processed B. Over the past five years there has been a substantial decline in
Tuesday and Wednesday, not Friday. Sulfur dioxide [(B), (E)] is the number of accounting school students, while electronic
not processed on Friday. books have remained fairly stable.
C. In the five years immediately preceding the price surge, there
was a substantial decline in the number of accounting school
17. E: Liquid hydrogen is only processed on Monday, students requiring text books, while electronic book
Wednesday, and Friday. Of these three days, on Wednesday subscriptions remained fairly stable.
liquid hydrogen and an additional three chemicals-xenon, D. Many electronic publishers have recently begun cutting back
oxygen, and sulfur dioxide-may be processed. On Monday, on subscriptions of accounting school text books.
liquid hydrogen plus sulfur dioxide and methane can be E. In almost every publishing company, there has been an
processed. On Friday, liquid hydrogen plus xenon and methane increase in the number of accounting school texts available in the
can be processed. past few years.
18. E: With testing in progress, results are not yet definitive. The
2. A pesticide producing company states that their unused
passage never indicates that preliminary results apply only to a
pesticide that gets dumped does not pose a threat to the aquatic
small minority (A). Six cups of coffee daily is not necessarily
life in the surrounding area. If this is correct, then why have
higher than what many people drink, which the passage also
local fish been dying in this region? Due to the fact that the
does not identify (B). While generalizability of animal results to
pesticide company is not located in a highly fish-populated
humans is doubted by many (C), the media relations person did
area, they implicitly admit that the pesticides they produce are
NOT cite this objection, OR limited application of rat studies to
relatively dangerous to the nearby aquatic life.
human birth defects (D).
Of the following statements listed below, which one would be
19. C: It helps to make a chart/table with cells for first name,
most likely to weaken the argument of the author if it were
surname, place in line, and food, first filling in the information
true?
given and then identifying the missing pieces through matching
and elimination: Jessica Trotter was first in line and bought
Cumberland sausage. Maggie Bore was second and bought pork A. The possibility of pesticides filtering into the local water
pie. Annie Hazlitt was third and bought sliced ham. Lily Piggott region was underestimated in the past.
B. Funds for environmental company cleanup, which concern
waste dumps that are poorly run, are reserved for rural regions 6. The quarterly food inspection performed by the local health
only. team observes the customer reactions to fast food restaurants
C. It would be pointless to locate chemical dumps where they and family dining restaurants. However, during each
would be most harmful, unless they can be proven 100-percent inspection they discover that there are more reports of food
safe. poisoning found in the family dining restaurants than in the
D. Dumps that are located in areas without large fish populations fast food restaurants.
have fewer government interventions and are also less
expensive. Of the following statements, which one best clarifies the
E. City people are most probable to sue the company if the apparent paradox?
dumps cause them health problems.
A. Customers are most likely to connect the illness they've
3. China wants to avoid financial collapse of their economy. In experienced to their most recent meal if the illness has suddenly
order to do this, China must raise their gross national product struck all of the people they ate with.
rate by 33 percent. China's economy is structured so that if the B. Customers complain less about the food poisoning they
33 percent increase in GNP is reached, then it is possible for a experience in fast food restaurants because they expect it.
50 percent GNP increase. C. More people choose fast food restaurants over family dining
restaurants.
Of the following statements listed below, which one must be D. Food poisoning cases are not related to the time customers ate
true if is it to be believed that the above statements are also at the family dining restaurants, or to the number of people who
true? all ate the same meal.
E. The family dining restaurants microwaves certain food items
A. If China's 50 percent increase in GNP is unattainable, then its instead of cooking them on the stove.
economy will collapse.
B. China's GNP will not have a 50 percent increase if its 7. We are well aware that there are warning signs concerning
economy falls. massive climate changes, and that these climate changes are
C. The economy of China will not fall if it can obtain an reducing plant life. Many hopeful crop growers believe that
increased GNP of 50 percent. there will not be an overall negative effect on the plant growth
D. A 17 percent GNP increase will be unattainable if China population due to the fact that rainfall should not be altered
continues to suffer national conflict. because of the climate changes. However, for the average
E. A 71 percent increase is possible if the 33 percent brink is plant, it is because of the climate change that agricultural
achieved, and the 50 percent GNP increase is attainable. technology has an overall yield in annual fluctuation.
4. Estelle states: When I went fishing the other day, every fish On which of the following assumptions are these claims
that I caught was a salmon, and every salmon I saw I caught. based?
Of the following statements listed below, which one can be A. There is not an accurate way to predict a climate change.
concluded from the observations of Estelle? B. If patterns of rainfall began to shadow the climate changes,
there would be supplementary damaging effects.
A. Salmon was the only fish that Estelle saw while she was C. Improved yields grow highly unlikely if technology is
fishing. significantly influential in spite of climate change.
B. While Estelle was fishing, she caught no fish other than D. Rainfall patterns are not as predictable as patterns of
salmon. temperature.
C. In the area that Estelle fished, there were no other fish. E. Plant life is threatened more from cool temperatures than
D. All of the fish that Estelle saw she caught. warm ones.
E. Estelle did not see any other fish while she was fishing.
8. Different life forms such as animals and plants are known to
5. Either protesters must have restrictions placed on them, or have certain breeds that have extended life spans compared to
particular revolutionary issues that arise in society will be used that of humans. Due to this fact, scientists pass away before
to destroy the country. Because allowing the occurrence of the they are able to study the complete life cycle of these certain
revolutionary outcome is out of the question, we must restrict breeds. This being the case, a single breed may be inferred by
protesters. observation over various life stages. Geology or astronomy can
be applied to using the same method. Scientists can also use
this method to learn about desert evolution and rock
The above conclusion is unsteady due to the fact that...
formations.
A. Protesters do not really want to demolish the country.
B. There is too much emphasis placed on the importance of Concerning this passage, what assumption can be made
protesting. evident?
C. An accommodation is failed to be considered between both
alternatives. A. The average subject uses the same ideals of observation.
D. The reasons for protesting have not been defined. B. Certain endangered breeds need to be studied more before
E. Protesters are, in truth, a real threat to the country's survival. they become extinct.
C. Various stage developments of different breeds are available
to scientists as examples for both study and observation.
D. Through the use of today's study technique there are many B. Rackets that are strengthened by oak wood are used
breeds in society that cannot be properly studied in the exclusively in Wilson's new rackets.
environment. C. Oak-wood-strengthened rackets help to make tennis rackets
E. There are different techniques that scientists can use that are durable and stronger, allowing the player to make powerful
available in modern society. swings.
D. With Wilson's rackets, you will improve your tennis playing.
9. A rich businessman runs a prosperous company. He is E. The status achieved by the past three consecutive women's
disappointed in his two children, Violet and Hazen, because he U.S. tennis championships was due to the assistance of Wilson's
believes that neither of them presents the potential of having rackets.
the ability to take control of his company. He thinks that both
of his children lack common sense. 12. My family doctor said that he would be performing a blood
test on me when I visit him today. I know I will feel pain today.
This belief formulates from the opinion that...
The above argument depends on which one of these
A. Violet and Hazen are ignorant to the experience of controlling assumptions?
the company.
B. Even a person who is not brilliant can control a company if A. The use of a needle always causes pain in the patient.
she or he has been able to obtain an MBA. B. The doctor will have a hard time finding the patient's vein.
C. In order to run a company, a person needs common sense. C. In the past, this patient has experienced pain at the family
D. If Hazen showed any sign of common sense, he would have doctor.
the ability to aid Violet in controlling the company. D. The needle will leave a bruise.
E. A committee with an average of three trained personnel could E. The doctor will have to try different needles to perform the
assist either Violet or Hazen in controlling the company. test.
10. The regulation that is proposed for dental insurance will 13. An English school teacher requested her students to try and
contribute only small assistance to patients in the reduction of write children's stories that are relevant to their everyday lives.
costs for regular dental care. Although the bill limits the The idea would be to give their stories a quality of "real life".
amount that the dentist can charge for a regular visit, it does One of the students decided to base her story upon one of the
not limit the expense amount that they can charge if a patient fictional characters from her favorite novel.
is in need of a special procedure, and it doesn't place any limits
on the amount of times that the dentist may see a patient for Of the following criteria listed below, which would make the
the same occurring problem. This being the case, instead of the most logical sense as criticism against the student's choice for
patient being charged once, the dentist can bill the patient her children's story?
numerous times, and the total costs will not reduce.
A. The writing techniques that are successful for one writer are
The above argument is opposed to the new legislation based frequently unsuccessful for another.
on...
B. A story based entirely on the writer's knowledge of characters
A. Identifying a loophole in the regulation proposal that allows from another novel is not likely to include the writer's personal
dentists to charge patients the same amount of money on a feelings of real life.
continuing basis. C. The author of her favorite novel would not allow the student
B. Implication that the regulation of dental care is impossible. permission to use their character.
C. The suggestion that procedures which are specialized are D. Children's story writing requires examination of the self in
frequently done when a simple, less-expensive procedure would order to develop innovative and original ideas.
work equally as well. E. A writer should concentrate on developing themes of teaching
D. The suggestion that regular dental visits are much more and significance, instead of simply aiming for prominence.
expensive than specialized procedures.
E. The suggestion that patients cannot distinguish between what 14. A monopoly is distinguished through the decline or lack of
appropriate dental care expenses there are for any known dental competition. The MANG Company recognizes that its
problem. operations are within a competitive field.
11. The past three consecutive women's U.S. tennis champions Of the following conclusions, which one may be an
have all changed to Wilson's new line of tennis rackets, implication of the above statement?
exclusively made of oak wood for greater strength and
durability. If this is the case, don't you think it's time to A. A one-seller market is the definition of a monopoly.
improve your tennis swing and trade your old racket in for a B. There is no family competition in the MANG Company.
Wilson? C. The MANG Company's focus is non-monopolistic.
D. The MANG Company operates within a service industry.
Which of the following claims is not made and cannot be used in E. The MANG Company is owned publicly.
conclusion to the above advertisement?
Refer to the following passage for questions 16 and 17: What is the assumption made by this ad?
While traveling to Japan, a low-ranking US ambassador asked a A. It's not hard to tell domestic water from imported water based
Japanese official why Japanese people were so inscrutable. The on its flavor.
official looked calm and friendly, responding in a gentle voice B. The majority of spring water is bottled at its source.
that he much preferred to think upon his race as inscrutable than C. Restrictions on importing and customs duties make the price
of his race as wanting in perspicacity such as in Americans. of imported water higher.
D. Spring water tastes best when it's served from a decanter.
16. Which of the following statements best describes the E. Some people purchase imported spring water instead of
Japanese official's comment? domestic as a status symbol.
A. All people are inscrutable, not just the Japanese. 21. Priest: Do you speak to the devil and follow his biddings?
B. Most Americans don't understand Japanese culture.
C. What a person lacks in perception may be a result of the Parishioner: Yes.
carelessness of the observer, instead of the obscurity within the
object being observed. Priest: You must be lying. Nobody who is in league with the devil
D. The Japanese distrust American ambassadors. tells the truth.
E. If the East and West are ever to understand one another, there
will need to be a much better cultural understanding.
Why can the priest's behavior be considered paradoxical?
17. Which of the following words best describes both the
A. He accused the parishioner of being in league with the devil,
attitude and the response made by the Japanese official?
but he later changed his story.
B. He relied upon the answer of the parishioner in order to reject
A. Fearful his response.
B. Emotional C. His behavior was entirely within accordance with religious
C. Angry law, but he was accusing the parishioner of violating that law.
D. Indifferent D. While he is questioning the parishioner about possible
E. Compassionate association with the devil, he doesn't actually believe in such a
thing.
18. One day, a poet was requested to interpret an especially E. He was the one who asked the question, but he refused to
peculiar and obscure passage within one of his own poems. His accept the answer.
response was "at the time that I was writing that particular
verse, only God and myself knew its meaning. Now, it is only Answers and Explanations
God who knows."
1. C: If e-book subscriptions remained stable in the five
What does the poet mean by his answer? preceding years while textbook purchases declined, this would
support the explanation for textbook price increases in response
A. God is much wiser than people are. to lower textbook sales. A publication backlog (A) would not
B. Most people can't understand poetry. raise prices. Student numbers declining (B) does not justify
C. Poets don't often know where their creative inspiration comes raising textbook prices when e-book sales did not decline.
from. Electronic publishers would not reduce textbook subscriptions
D. Great poems are inspired by a muse. (D), since they don't publish these. More available textbooks (E)
E. The poet has forgotten the meaning of his own verse. should lower, not raise, prices.
2. D: The statement that dumping in lower-fish areas is less 9. C: He believes neither child can take control of his company
regulated and less expensive weakens the argument that dumping because of his opinion that they "lack common sense."
there implies the dumped pesticide is dangerous, by supplying Therefore, he believes a person needs common sense to run a
two other motivations for dumping there. Previous company. That his children are "ignorant to the experience of
underestimation of danger (A) does not weaken and could controlling the company" (A) is not indicated as his opinion.
support the argument for implicitly admitting danger. The Nothing is mentioned about obtaining an MBA (B). He thinks
passage does not identify the area as rural (B). (C) contradicts both children lack common sense, not one (D). Committee
itself, making no sense. The passage does not identify the area as assistance (E) is never mentioned.
urban (E).
10. A: The argument identifies the loophole of unlimited special
3. A: The passage states China must raise its GNP by 33% to procedure expenses, and unlimited repeat visit charges for the
avoid economic collapse; if 33% is reached, 50% is possible. If same problem. It does not imply dental care cannot be regulated
50% is impossible, 33% was not reached; the economy will (B). Substituting specialized procedures for less-expensive
collapse. China's economy will fall without GNP increase, not procedures (C) is never suggested. The passage never suggests
vice versa (B); and by 33%, not 50% (B), (C). National conflict that regular visits cost more than specialized procedures (D), or
is not mentioned; neither is 17% (D) or 71% GNP increase (E). that patients cannot distinguish appropriate expenses (E).
4. B: If every fish Estelle caught was a salmon, all she caught 11. E: Champions "have all changed to" Wilson's new rackets;
were salmon. Salmon were not necessarily the only fish she saw they did not win past championships with them. Champions are
(A); she could have seen but not caught other fish. Likewise knowledgeable about tennis and their equipment (A): the ad uses
there were not necessarily no other fish there (C); she just didn't their choosing Wilson's as an example to follow. It indicates
catch them. Estelle caught all the salmon she saw, not all the fish exclusive use of oak in Wilson's new line (B). Durability and
she saw (D). (E) is the same as (A) strength are named as oak's benefits; from the suggestion these
will "improve your tennis swing," we can infer "powerful
5. C: The range of compromises between destroying the country swings" (C) and "you will improve your tennis playing" (D).
and restricting protesters is ignored. Some protesters do not want
to destroy the country (A), but some may. Therefore, emphasis 12. A: To know one will feel pain from a blood test, we must
on protesting's importance does not necessarily weaken the assume needles always cause this patient pain. Having trouble
conclusion (B). Defining reasons for protesting (D) is not finding the vein (B) is not the only cause of needle pain.
required to support the conclusion, which is based on protesters Experiencing pain with this doctor before (C) does not guarantee
destroying the country regardless of the issues. That protesters s/he always will. Leaving a bruise (D) does not always mean the
are truly threats (E) strengthens, not weakens, the conclusion. needle hurt. Having to try different needles (E) is not the only
cause of pain.
6. A: More diners eating together are more likely to notice all
subsequently experiencing food poisoning. Customers expecting 13. B: The student's real-life experience is unlikely to show in
food poisoning would not eat at fast food restaurants (B). Even if writing about fictional characters. Different writing techniques'
expecting more chance of it, they would still report occurrences. variable success among writers (A) is not criticism of her choice.
More people choosing fast food restaurants (C) would increase Author permission (C) is irrelevant to her choice's
the probability of food poisoning complaints from fast food, not inappropriateness. Self-examination is important to originality
family dining, restaurants. Choice (D) simply contradicts (A). (D), but misses the point: relevance to students' everyday lives,
Microwaving (E) is not associated with food poisoning. not originality, was assigned. Instructive, significant themes'
precedence over fame (E) is irrelevant to her choosing fictional
7. B: Although hopeful growers believe climate changes "should characters to describe her real-life experiences.
not" change rainfall, these cause annual agricultural yield
fluctuations. This claim proceeds from the assumption that since 14. C: Monopoly means declining/ therefore, lacking
yields already fluctuate from climate change, additional rainfall competition. By recognizing its field is competitive, the
change would exacerbate fluctuations. That climate change company accepts competition and has a non-monopolistic focus.
cannot be accurately predicted (A) is not claimed. Technology's Monopoly's definition as a one-seller market (A) is not implied:
significant influence despite climate change makes improved the statement indicates only that monopoly is "distinguished
yields likely, NOT "highly unlikely" (C). That rainfall is less through the decline or lack of competition." Family competition
predictable than temperature (D), or cool temperatures more (B), service industry (D), and public ownership (E) are never
threatening (E), are not claims. implied in the statement.
8. C: Scientists can observe and study various life stages of 15. A: If society believes local AA facilities are unsuccessful,
breeds too long-lived to study their full life span during society will not fund them. Cost exceeding local funding (B)
scientists' lifetimes. The passage indicates subjects' using the does not weaken the argument: "society" does not necessarily
same observational methods, but using the same ideals (A) mean local (and more likely means federal funding.)
cannot be assumed. Choice (B) is true, but not indicated in the Neighborhood group support (C) would strengthen, not weaken,
passage. Today's techniques preventing studying many breeds in the argument for local facilities. Maintenance cost (D)
the environment (D) is nowhere suggested. Various scientific strengthens the argument that if society wants to force facilities
techniques are available (E), but this cannot be inferred from the on communities, society should pay for them. Alcoholics'
passage. treatment resistance (E) is irrelevant to forcing/funding facilities.
16. C: The official attributed Americans' perception of Japanese ways they have influenced their husbands. Other First Ladies
as "inscrutable" to American lack of discernment, not Japanese have made the history books on their own.
inaccessibility. He never suggested all people are inscrutable
(A). He did not say that most Americans don't understand At least two First Ladies, Bess Truman and Lady Bird Johnson,
Japanese culture specifically (B), but that Americans lack made it their business to send signals during their husbands'
understanding generally. He never said the Japanese mistrust speeches. When Lady Bird Johnson thought her husband was
American ambassadors (D). For the offensive "inscrutable" talking too long, she wrote a note and sent it up to the platform.
stereotype, he returned a reciprocal opinion of Americans as It read, "It's time to stop!" And he did. Once Bess Truman didn't
unperceptive, rather than calling for improved cultural like what her husband was saying on television, so she phoned
understanding (E). him and said, "If you can't talk more politely than that in public,
you come right home."
17. E: The official's response and attitude were compassionate,
indicated by his "calm and friendly" look and "gentle voice." His Abigail Fillmore and Eliza Johnson actually taught their
words also demonstrated a sense of humor. The description of husbands, Millard Fillmore and Andrew Johnson, the thirteenth
his behavior is not fearful (A), emotional (B), angry (C), or and seventeenth Presidents. A schoolteacher, Abigail eventually
indifferent (D). While he deflected a racial stereotype of the married her pupil, Millard. When Eliza Johnson married
Japanese by introducing a corresponding one of Americans, he is Andrew, he could not read or write, so she taught him herself.
not described as doing so with fear, emotion, anger, or
indifference.
It was First Lady Helen Taft's idea to plant the famous cherry
trees in Washington, D. C. Each spring these blossoming trees
18. E: The poet humorously couches the fact that he forgot what attract thousands of visitors to the nation's capital. Mrs. Taft also
he meant by saying only he and God knew at the time, and now influenced the male members of her family and the White House
only God knows-i.e., now he himself does not know. He is not staff in a strange way: she convinced them to shave off their
saying God is wiser (A), but remembers better. He never says beards!
people can't understand poetry (B); he refers only to himself not
knowing/remembering his meaning. He refers only to the verse's Shortly after President Woodrow Wilson suffered a stroke, Edith
meaning, not its inspiration (C), (D). Wilson unofficially took over most of the duties of the
Presidency until the end of her husband's term. Earlier, during
19. B: This is a syllogism. Syllogisms have three parts: major World War I, Mrs. Wilson had sheep brought onto the White
premise, minor premise, conclusion. Syllogism uses deductive House lawn to eat the grass. The sheep not only kept the lawn
reasoning, reducing general information (all birds have beaks, all mowed, but provided wool for an auction sponsored by the First
sparrows are birds) to infer a more specific conclusion (all Lady. Almost $100,000 was raised for the Red Cross.
sparrows have beaks). Inductive reasoning (D), the opposite of
deductive, accumulates specific facts to form generalizations
Dolly Madison saw to it that a magnificent painting of George
(A). No special training (C) was required to make this
Washington was not destroyed during the War of 1812. As the
conclusion. The argument involves no ambiguity (E): it is not
British marched toward Washington, D. C., she remained behind
open to multiple interpretations or unclear. to rescue the painting, even after the guards had left. The
painting is the only object from the original White House that
20. E: The ad assumes some people find imported spring water a was not burned.
status symbol by mentioning their embarrassment over serving
domestic. It assumes it is hard to tell these apart by flavor (A):
One of the most famous First Ladies was Eleanor Roosevelt, the
"You'll never taste the difference...". There is no assumption wife of President Franklin D. Roosevelt. She was active in
most spring water is bottled at its source (B). The only reason the political and social causes throughout her husband's tenure in
ad assumes for higher prices is status, not regulations (C). The
office. After his death, she became famous for her humanitarian
decanter conceals water's domestic origin rather than improving
work in the United Nations. She made life better for thousands of
its taste (D).
needy people around the world.
Reading Main Idea Practice Questions Read the passage below and answer question 2.
Before any fry makes it to the people who eat at these popular Another basic principle was brought into law because certain
restaurants, it must pass many separate tests. Fail any one of influential people in England had managed to obtain monopoly
these tests and the potato is rejected. To start with, only Russet control over such age-old products as salt, and had begun
Burbank potatoes are used. These Idaho potatoes have less water charging as much as the people could tolerate. The public outcry
content than other kinds, which can have as much as 80 percent became so great that the government was forced to decree that
water. Once cut into "shoestrings" shapes, the potatoes are partly monopoly rights could be awarded only to those who created or
fried in a secret blend of oils, sprayed with liquid sugar to brown introduced something really unique. These principles are the
them, steam dried at high heat, then flash frozen for shipment to mainstays of the modern patent system in the United States.
individual restaurants.
In colonial times, patent law was left up to the separate states.
Before shipping, every shoestring is measured. Forty percent of a The inconsistency, confusion, and unfairness that resulted clearly
batch must be between two and three inches long. Another 40 indicated the need for a uniform patent law, and the men who
percent has to be over three inches. What about the 20 percent drew up the Constitution incorporated one. George Washington
that are left in the batch? Well, a few short fries in a bag are signed the first patent law on April 10, 1790, and less than four
okay, it seems. months later the first patent was issued to a man named Samuel
Hopkins for a chemical process, an improved method of making
potash for use in soapmaking.
So, now that you realize the enormous size and value of the
potato crop, you can understand why most people agree that this
part of the food industry is no "small potatoes." In 1936 the Patent Office was established as a separate bureau.
From the staff of eight that it maintained during its first year of
operation, it has grown into an organization of over 2,500 people
2. What is the main idea of this passage?
handling more than 1,600 patent applications and granting over
1,000 every week.
A. Potatoes from Ireland started the Potato Revolution.
B. The average American eats 50 pounds of potatoes a year.
C. French fries are made from potatoes. The Patent Office in Washington, D. C. is the world's largest
D. Potatoes are a key vegetable in America. library of scientific and technical data, and this treasure trove of
information is open for public inspection. In addition to more
E. The various terms for potatoes have a long history.
than 3 million US patents, it houses more than 7 million foreign
patents and thousands of volumes of technical literature.
Read the passage below and answer question 3. Abraham Lincoln patented a device to lift steam vessels over
river shoals, Mark Twain developed a self-pasting scrapbook, Beavers think nothing of building a dam more than 200 feet
and millionaire Cornelius Vanderbilt invented a shoe-shine kit. long. One dam in Montana was more than 2,000 feet long. The
largest one ever seen was in New Hampshire: it stretched 4,000
A patent may be granted for any new and useful process, feet, and made a lake large enough to hold 40 beaver homes.
machine, article of manufacture, or composition of matter (a
chemical compound or combinations of chemical compounds), So beavers do build good dams. But they don't always build
or any distinct and new variety of plant, including certain them in the right places. They just don't plan. They will build a
mutants and hybrids. dam across the widest part of the stream. They don't try to find a
place where the stream is narrow. So a lot of their hard work is
The patent system has also helped to boost the wages of the wasted.
American worker to an unprecedented level: he can produce
more and earn more with the computer, adding machines, drill Beavers should learn that it's not enough to be busy. You have to
press or lathe. Patented inventions also help keep prices down by know what you're doing, too. For example, there was one
increasing manufacturing efficiency and by stimulating the Oregon beaver that really was a worker. It decided to fix a leak
competition that is the foundation of our free enterprise system. in a man-made dam. After five days of work it gave up. The leak
it was trying to block was the lock that boats go through.
The decades of history have disclosed little need for
modification of the patent structure. United States patent laws, 4. What is the main idea of this passage?
like the Constitution from which they grew, have stood the test
of time well. They encouraged the creative processes, brought A. Beavers may be hard-working animals, but they don't always
untold benefits to society as a whole, and enabled American choose the most efficient mechanisms.
technology to outstrip that of the rest of the civilized world. B. Beavers are excellent dam builders.
C. New Hampshire was the site of the largest beaver dam.
3. What is the main idea of this passage? D. Beavers are well-developed tree cutters.
E. Beavers are poor surveyors of aquatic environments in some
A. The patent system encourages free enterprise. cases.
B. The Constitution protects the patent system.
C. The patent system in England has been influential in Read the passage below and answer question 5.
American patent development.
D. Patents are important tools for inventors. The raisin business in America was born by accident. It
E. Patented inventions protect the inventor, free enterprise, and happened in 1873 in the San Joaquin Valley of California. Many
the creative process. farmers raised grapes in this valley. That year, just before the
grape harvest, there was a heat wave. It was one of the worst
Read the passage below and answer question 4. heat waves ever known. It was so hot that the grapes dried on the
vines. When they were picked, California had its first raisin crop.
Most people think that it's fine to be "busy as a beaver." Little do
they know. Beavers may work hard, but often they don't get very People were surprised to find how good raisins were. Everybody
much done. wanted more. So the San Joaquin farmers went into the raisin
business. Today, of course, they do not let the grapes dry on the
Beavers are supposed to be great tree cutters. It is true that a vines. They treat them with much more care.
beaver can gnaw through a tree very quickly: A six-inch birch
takes about 10 minutes. But then what? Often the beaver does In late August the grapes start to ripen. They are tested often for
not make use of the tree. One expert says that beavers waste one sweetness. The growers wait until the sugar content is twenty-
out of every five trees they cut. one percent. Then they know the grapes are ripe enough to be
picked.
For one thing, they do not choose their trees wisely. One bunch
of beavers cut down a cottonwood tree more than 100 feet tall. Skilled workers come to the vineyards. They pick the grapes by
Then they found that they could not move it. hand in bunches. The workers fill their flat pans with grapes.
They gently empty the pans onto squares of paper. These squares
In thick woods, a tree sometimes won't fall down. It gets stuck in lie between the long rows of vines. They sit in the sun.
the other trees. Of course, the beaver doesn't think to cut down
the trees that are in the way. So a good tree goes to waste. Here the grapes stay while the sun does its work. It may take two
weeks or longer. The grapes are first dried on one side. When
Some people think that beavers can make a tree fall the way they they have reached the right color, they are turned to dry on the
want it to. Not true. (In fact, beavers sometimes get pinned under other side. The grapes are dried until only fifteen percent of the
a falling tree.) When beavers cut a tree near a stream, it usually moisture is left. Then they have turned into raisins.
falls into the water, but they do not plan it that way. The fact is
that most trees lean toward the water to start with. The raisins are rolled up in the paper on which they have dried.
Trucks take them from the fields. They are poured into big boxes
Now what about dam building? Most beaver dams are wonders called sweatboxes. Each box holds 160 pounds of raisins. Here,
of engineering. The best ones are strongly built of trees, stones, any raisins that are too dry take moisture from those that have
and mud. They are wide at the bottom and narrow at the top. too much. After a while, they are all just moist enough.
The big boxes are trucked next to the packaging plant. They are with his school records from Cambodia, and they were
emptied onto a conveyor belt that shakes the raisins gently. This impressed with his bravery. Finally, in 1980, he was able to
knocks them from their stems. A blast of air whisks the stems study at Columbia University in New York City. After finishing
away. The water bath is next. Then the plump brown raisins have his studies at Columbia, Siv took a job with the United Nations.
a last inspection. They are again checked for moisture and sugar. He married an American woman and became a citizen. After
Then they go on a belt to packing machines. Here they are several more years, he felt that he was very much a part of his
poured into packages, which are automatically weighed and new country.
sealed. The raisins are now ready for market.
In 1988, Siv was offered a job in the White House working for
5. What is the main idea of this passage? President Ronald Reagan's closest advisors. It was a difficult job,
and he often had to work long hours. However the long hard
A. The creation of raisins in America was an accident. work was worth it, because Siv got the opportunity to help
B. The process of raisin development requires multiple steps. refugees in his work.
C. Raisins on the grocery store shelf undergo a brief
fermentation process. 6. What is the main idea of this passage?
D. Raisins are cleaned thoroughly at the packing plant.
E. California has been the leader in American raisin A. Persistence and courage are global ideas.
development. B. Siv covered a large area during his life.
C. Siv persevered to escape from Cambodia.
Read the passage below and answer question 6. D. Siv overcame numerous challenges to come to America and
help others.
In 1976, Sichan Siv was crawling through the jungle, trying to E. Siv persevered to become an American citizen.
escape from Cambodia. By 1989, however, Siv was working in
the White House in Washington D. C., as an advisor to the Read the following passage and answer question 7.
President of the United States. How did this strange journey
come about? When you want to hang the American flag over the middle of a
street, suspend it vertically with the blue field (called the union)
Like millions of Cambodians, Siv was a victim of a bloody civil to the north and east-west street. When the flag is displayed with
war. One of the sides in this war was the Cambodian another banner from crossed staffs, the American flag is on the
government. The other was a group called the Khmer Rouge. right. Place the staff of the American flag in front of the other
When the Khmer Rouge won the war, the situation in Cambodia staff. Raise the flag quickly and lower it slowly and respectfully.
got worse. Many people were killed, while others were forced When flying the flag at half-mast, hoist it to the top of the pole
into hard labor. Sometimes entire families were murdered. for a moment before lowering it to mid-pole. When flying the
American flag with banners from states or cities, raise the
Siv came from a large family that lived in the capital of nation's banner first and lower it last. Never allow the flag to
Cambodia. After finishing high school, Siv worked for a while touch the ground.
with a Cambodian airline company. Later, he taught English.
After that, he took a job with CARE, an American group that 7. What is the main idea of this passage?
was helping victims of the war.
A. The American flag is the symbol of American freedom.
Siv had hoped to leave Cambodia before the Khmer Rouge took B. The American flag has fifty stars.
over the country. Unfortunately, he was delayed. As a result, he C. Placing the American flag inappropriately will draw
and his family were taken from their homes and forced to labor government intervention.
in rice fields. Eventually, Siv managed to escape. He rode an old D. American flag should be flown differently in certain
bicycle for miles, trying to reach Thailand where he would be situations.
free and safe. For three weeks, he slept on the ground and tried E. The flag should be lowered quickly and respectfully.
to hide from the soldiers who were looking for him. Caught at
last, he was afraid he would be killed. Instead, he was put into a Read the following passage and answer question 8.
labor camp, where he worked 18 hours each day without rest.
After several months, he escaped again, and this time he made it.
What if someone told you about a kind of grass that grows as tall
The journey, however, was a terrifying one. After three days of as the tallest trees? A grass that can be made as strong as steel?
staggering on foot through mile after mile of thick bamboo, Siv
A grass from which houses, furniture, boats, and hundreds of
finally made his way to Thailand.
other useful things can be made? A grass that you would even
enjoy eating? Would you believe that person? You should, for
Because he had worked for an American charity group, Siv that grass is bamboo, the "wood" of 1,001 uses.
quickly found work in a refugee camp. Soon he was on his way
to the United States. He arrived in June of 1976 and got a job-
Bamboo may look like wood, but it is part of the family of plants
first picking apples and then cooking in a fast-food restaurant.
that includes wheat, oats, and barley. It is a kind of grass. This
Siv, however, wanted more than this: he wanted to work with
grass is not just a material for making useful products. Young
people who, like himself, had suffered the hardship of leaving
bamboo is eaten, often mixed with other vegetables, in many
their own countries behind. Siv decided that the best way to Asian foods.
prepare for this kind of work was to go to college. He wrote
letters to many colleges and universities. They were impressed
Bamboo grows in many parts of the world. In the United States it For most of the runners, however, the challenge of the race is the
grows in an area from Virginia west to Indiana and south to main reason for coming. On the first day, for example, they run
Florida, Louisiana, and Texas. Most bamboo, however, is found 15 miles across a desert of sand, rocks, and thorny bushes. Few
in warm, wet climates, especially in Asia and on the islands of runners finish the day without blistered and raw feet. Because
the South Pacific Ocean. they are allowed less than nine quarts of water during each day
of the race, they also suffer from a lack of water. Most of all,
In most Asian countries, bamboo is nearly as important as rice. they are exhausted when they arrive at the campsite for the night.
Many Asians live in bamboo houses. They sit on bamboo chairs
and sleep on bamboo mats. They fence their land with bamboo The second day, the runners awaken at 6:00 a.m. Within a few
and use it to cage their chickens and pigs. hours, it is 100 degrees Fahrenheit, but the runners do not
hesitate. They must cover 18 miles that day. That night, they
Bamboo is used to build large buildings as well as homes. When rest. They must be ready for the next day's run.
it is glued in layers, it becomes as strong as steel. On some
islands in the South Pacific, bamboo is even used for water On the third day, the runners must climb giant sand dunes-the
pipes. This extraordinary material has many other uses. It is used first they have faced. Dust and sand mix with the runners' sweat.
to make musical instruments such as flutes and recorders. Paper Soon their faces are caked with mud. After 15 miles of these
made from bamboo has been highly prized by artists for conditions, the runners finally reach their next camp.
thousands of years.
The race continues like this for four more days. The fourth and
Bamboo is light and strong, and it bends without breaking. It is fifth days are the worst. On the fourth day, the runners pass
cheap, floats on water, almost never wears out, and is easy to through a level stretch and a beautiful, tree-filled oasis, but then,
grow. Nothing else on earth grows quite so fast as bamboo. At on this and on the next day, they cross more than 21 miles of
times you can even see it grow! Botanists have recorded growths rocks and sand dunes. The temperature soars to 125 degrees
of more than three feet in just 24 hours! Bamboo is hollow and Fahrenheit, and many runners cannot make it. Helicopters rush
has a strong root system that almost never stops growing and fallen runners to medical help. Runners who make it to the end
spreading. In fact, only after it flowers, an event that may happen of the fifth day know that the worst is over.
only once every 30 years, will bamboo die.
On the sixth day, heat and rocks punish the racers terribly. In the
There are more than 1,000 kinds of bamboo. The smallest is only Valley of Dra, the wind picks up and, as the desert heat is thrust
three inches tall and one-tenth of an inch across. The largest is against them with great force, they grow more and more
more than 200 feet in height and seven inches in diameter. No exhausted.
wonder, then, that the lives of nearly half the people on earth
would change enormously if there were no longer any bamboo. The seventh day is the last, with only 12 miles to be covered.
No wonder, too, that for many people, bamboo is a symbol of The dusty, tired, blistered runners set out at daybreak. Near the
happiness and good fortune. finish line, children race along with the runners, for everybody
has caught the excitement. The ones who have run the whole
8. What is the main idea of this passage? marathon know they have accomplished what most people could
not even dream of. "During the hard moments," says one
A. Bamboo has at least 2,000 uses. contestant who has raced here twice, "I'd think, 'Why am I here?'
B. Bamboo grows at an amazing rate and is found primarily in Then I'd realize I was there to find my limits."
Asia.
C. Bamboo is an amazing grass that can be used in multiple 9. What is the main idea of this passage?
ways.
D. There are at least 1,000 types of bamboo. A. The Marathon of the Sands race tests the limits of human
E. Bamboo could be considered a flower in some cases. endurance.
B. The runners run at their own paces.
Read the following passage and answer question 9. C. The race causes the strong to stumble and the weak to not
finish.
Every year since 1986, some of the world's most daring runners D. The seventh day is the hardest day of the race.
have gathered in the desert of Morocco. They are there to take E. Every runner runs the race to find their human limits.
part in one of the most difficult races in the world. The Marathon
of the Sands, as it is called, covers over 125 miles of desert and Read the following passage and answer question 10.
mountain wilderness. The runners complete the course in fewer
than seven days, and they run with their food, clothing, and High in the Andes Mountains in Peru stands the ancient city of
sleeping bags on their backs. Machu Picchu. No one knows why this great city was built, nor
is it likely that anyone will ever know. Nevertheless, the deserted
The Marathon of the Sands was founded in 1986 by Patrick city of Machu Picchu is important for what it reveals about the
Bauer. His idea was to give the runners, who come from all over ancient Inca people of South America.
the world, a special kind of adventure. Most of the runners in this
race have found that they form deep friendships with the other The Incas once ruled a great empire that covered a large part of
runners during their days and nights in the desert. Facing terrible the South American continent. The empire was more than 500
heat and complete exhaustion, they learn much about themselves
and each other.
years old when the first Spanish explorers, looking for gold, the first paragraph states some First Ladies are remembered for
went to that continent in the 16th century. influencing their husbands, while others "...have made the
history books on their own." Not all First Ladies are described
The Incas were an advanced people. They were skillful engineers here as doing humanitarian work (A). No one First Lady is
who paved their roads and built sturdy bridges. They plowed the singled out as most important [(B), (C)].
land in such a way that rains would not wash away valuable soil,
and dug ditches to carry water into dry areas for farming. 2: D: The main idea is the importance of potatoes in America. It
never mentions Ireland or any Potato Revolution (A). (B) is both
Even though they did not know about the wheel, the Incas were incorrect-the passage states 100 lbs., not 50-and regardless of
able to move huge stone blocks-some as heavy as 10 tons-up the accuracy, is a detail, not the main idea. Readers already know
sides of mountains to build walls. The blocks were fitted so French fries are made from potatoes (C), a detail the passage
tightly, without cement of any kind, that it would be impossible assumes. Several various terms for potatoes are mentioned in the
to slip a knife blade between them! The walls have stood firm second paragraph, but their history (E) is never discussed.
through great storms and earthquakes that have destroyed many
modern buildings. 3. E: All three benefits of patents-inventor protection, free
enterprise, and the creative process-are given equal importance
The Incas were great artists, too. Today, Incan dishes and other in the passage. The other four choices each accurately identify
kinds of pottery are prized for their wonderful designs. Because individual ideas in the passage, but none incorporates all three
both gold and silver were in great supply, the Incas created parts of the main idea.
splendid objects from these precious metals.
4. A: Only this choice identifies the main idea, that beavers are
While it is true that the Incas had no written language, they kept hard-working but not always efficient. Each of the other choices
their accounts by using a system of knotted strings of various identifies one detail included in the passage, not the main idea.
lengths and colors. The sizes of the knots and the distances
between them represented numbers. 5. B: The multiple steps required in the process are outlined
throughout the passage. The introductory statement that the
At its height, the Incan Empire included as many as 30 million industry began by accident (A) is a detail, not the main idea.
people. The emperor ruled them with an iron hand. He told his Shelf fermentation (C) is never mentioned. A water bath is
subjects where to live, what to plant, how long they should work, mentioned, not thorough cleaning (D), and is a detail, regardless.
and even whom they could marry. Because he owned everything, California is only mentioned as the location of the first raisin
the emperor gave what he wished when he wished-and in the crop but never identified as raisin development's leader (E).
amount he wished-to his people.
6. D: The passage focuses on the many challenges overcome by
In 1533, Spanish explorers led by Francisco Pizarro murdered the subject, Siv, in particular, rather than mentioning any global
the emperor of the Incas. Earlier, the heir to the Incan empire had nature of his persistence and courage (A) or the size of the area
also been killed. The Incas, who had always been entirely he covered (B). His perseverance to escape (C) is true, but only
dependent on their emperor, now had no recognized leader. The part of the main idea, not mentioning his desire to help others.
Spaniards easily conquered the empire and plundered its riches. His attaining American citizenship is mentioned, but his
perseverance was not for this (E), but for escaping Cambodia
and helping other refugees.
Have the Incas disappeared from South America? Not at all. In
Peru alone, once the center of that great empire, 80 percent of the
20 million people are descendants of the Inca people. Evidence 7. D: The passage instructs how to fly the flag in different
of the Incan empire can be found in many other places in South situations. It never mentions the flag's symbolism (A) or its
America as well. Tourists can even visit Machu Picchu. The number of stars (B), or any government intervention (C). It states
remains of this ancient city still stand high in the mountains of the flag should be lowered slowly, not quickly (E).
Peru, an awesome tribute to this once powerful empire.
8. C: The many uses of bamboo, and the fact it is a grass, are the
10. What is the main idea of this passage? main focus. "1,001 uses" is a non-literal colloquial expression
meaning a great many; the passage never states factually that
bamboo has at least 2,000 (A). Bamboo's growth rate and its
A. The Incas once inhabited the ancient city of Machu Picchu.
B. Peru was the primary country of the Incas. location in many parts of the world, especially Asia (B), and the
number of types (D), and the fact that it occasionally flowers (E),
C. The Incan Empire can be found in ancient cities and was
are details supporting/informing the main idea.
plundered by the Spanish.
D. Spanish conquerors destroyed the Incan empire in the 13th
century. 9. A: Testing human endurance limits is illustrated in this
E. Machu Picchu was the capital of the Incan empire. description of a punishing marathon. It never mentions runners
running at their own paces (B). The race's challenges, and many
not finishing (C) are details informing the main point. The
Answers and Explanations seventh/last day is not the hardest (D); the fourth and fifth are
identified as worst. (Regardless of accuracy, this is also a detail,
1. D: The passage describes actions of various First Ladies as not the main idea.) One, not every (E), runner is quoted as
examples of their importance in American culture. That they are competing "to find my limits."
key supporters of the Presidents (E) is not the main idea because
10. C: This choice best summarizes the passage's main points. 3. The Pope divided New World lands between Spain and
Choice (D) incorrectly identifies the 13th century instead of the Portugal according to their location on one side or the other
16th century. The passage never indicates that Machu Picchu of an imaginary geographical line 50 degrees west of
was the capital of the Incan empire (E). Answers (A) and (B) are Greenwich that extends in a _________ direction.
details in support of the main idea.
A. north and south
Reading Comprehension Practice Test B. crosswise
C. easterly
Questions 1 through 7 refer to the following passage: D. south east
E. north and west
In the 16th century, an age of great marine and terrestrial
exploration, Ferdinand Magellan led the first expedition to sail 4. One of Magellan's ships explored the _________ of South
around the world. As a young Portuguese noble, he served the America for a passage across the continent.
king of Portugal, but he became involved in the quagmire of
political intrigue at court and lost the king's favor. After he was A. coastline
dismissed from service by the king of Portugal, he offered to B. mountain range
serve the future Emperor Charles V of Spain. C. physical features
D. islands
A papal decree of 1493 had assigned all land in the New World E. None of the above
west of 50 degrees W longitude to Spain and all the land east of
that line to Portugal. Magellan offered to prove that the East 5. Four of the ships sought a passage along a southern
Indies fell under Spanish authority. On September 20, 1519, ______.
Magellan set sail from Spain with five ships. More than a year
later, one of these ships was exploring the topography of South A. coast
America in search of a water route across the continent. This B. inland
ship sank, but the remaining four ships searched along the C. body of land with water on three sides
southern peninsula of South America. Finally they found the D. border
passage they sought near 50 degrees S latitude. Magellan named E. Answer not available
this passage the Strait of All Saints, but today it is known as the
Strait of Magellan. 6. The passage was found near 50 degrees S of ________.
One ship deserted while in this passage and returned to Spain, so A. Greenwich
fewer sailors were privileged to gaze at that first panorama of the B. The equator
Pacific Ocean. Those who remained crossed the meridian now C. Spain
known as the International Date Line in the early spring of 1521 D. Portugal
after 98 days on the Pacific Ocean. During those long days at E. Madrid
sea, many of Magellan's men died of starvation and disease.
7. In the spring of 1521, the ships crossed the _______ now
Later, Magellan became involved in an insular conflict in the called the International Date Line.
Philippines and was killed in a tribal battle. Only one ship and 17
sailors under the command of the Basque navigator Elcano
A. imaginary circle passing through the poles
survived to complete the westward journey to Spain and thus
B. imaginary line parallel to the equator
prove once and for all that the world is round, with no precipice
C. area
at the edge.
D. land mass
E. Answer not available
1. The 16th century was an age of great ______ exploration.
The following passage refers to questions 8 through 14.
A. cosmic
B. land
Marie Curie was one of the most accomplished scientists in
C. mental
history. Together with her husband, Pierre, she discovered
D. common man
radium, an element widely used for treating cancer, and studied
E. None of the above
uranium and other radioactive substances. Pierre and Marie's
amicable collaboration later helped to unlock the secrets of the
2. Magellan lost the favor of the king of Portugal when he atom.
became involved in a political ________.
Marie was born in 1867 in Warsaw, Poland, where her father
A. entanglement was a professor of physics. At an early age, she displayed a
B. discussion brilliant mind and a blithe personality. Her great exuberance for
C. negotiation learning prompted her to continue with her studies after high
D. problem school. She became disgruntled, however, when she learned that
E. None of the above the university in Warsaw was closed to women. Determined to
receive a higher education, she defiantly left Poland and in 1891
entered the Sorbonne, a French university, where she earned her A. Dejectedly
master's degree and doctorate in physics. B. Worried
C. Tearfully
Marie was fortunate to have studied at the Sorbonne with some D. Happily
of the greatest scientists of her day, one of whom was Pierre E. Irefully
Curie. Marie and Pierre were married in 1895 and spent many
productive years working together in the physics laboratory. A 13. Her _________ began to fade when she returned to the
short time after they discovered radium, Pierre was killed by a Sorbonne to succeed her husband.
horse-drawn wagon in 1906. Marie was stunned by this horrible
misfortune and endured heartbreaking anguish. Despondently A. misfortune
she recalled their close relationship and the joy that they had B. anger
shared in scientific research. The fact that she had two young C. wretchedness
daughters to raise by herself greatly increased her distress. D. disappointment
E. ambition
Curie's feeling of desolation finally began to fade when she was
asked to succeed her husband as a physics professor at the 14. Even though she became fatally ill from working with
Sorbonne. She was the first woman to be given a professorship at radium, Marie Curie was never _________.
the world-famous university. In 1911 she received the Nobel
Prize in chemistry for isolating radium. Although Marie Curie
A. troubled
eventually suffered a fatal illness from her long exposure to
B. worried
radium, she never became disillusioned about her work. C. disappointed
Regardless of the consequences, she had dedicated herself to D. sorrowful
science and to revealing the mysteries of the physical world.
E. disturbed
8. The Curies' _________ collaboration helped to unlock the The following passage refers to questions 15 through 19.
secrets of the atom.
Mount Vesuvius, a volcano located between the ancient Italian
A. friendly
cities of Pompeii and Herculaneum, has received much attention
B. competitive
because of its frequent and destructive eruptions. The most
C. courteous famous of these eruptions occurred in A.D. 79.
D. industrious
E. chemistry
The volcano had been inactive for centuries. There was little
warning of the coming eruption, although one account unearthed
9. Marie had a bright mind and a ______ personality. by archaeologists says that a hard rain and a strong wind had
disturbed the celestial calm during the preceding night. Early the
A. strong next morning, the volcano poured a huge river of molten rock
B. lighthearted down upon Herculaneum, completely burying the city and filling
C. humorous the harbor with coagulated lava.
D. strange
E. envious Meanwhile, on the other side of the mountain, cinders, stone and
ash rained down on Pompeii. Sparks from the burning ash
10. When she learned that she could not attend the university ignited the combustible rooftops quickly. Large portions of the
in Warsaw, she felt _________. city were destroyed in the conflagration. Fire, however, was not
the only cause of destruction. Poisonous sulfuric gases saturated
A. hopeless the air. These heavy gases were not buoyant in the atmosphere
B. annoyed and therefore sank toward the earth and suffocated people.
C. depressed
D. worried Over the years, excavations of Pompeii and Herculaneum have
E. None of the above revealed a great deal about the behavior of the volcano. By
analyzing data, much as a zoologist dissects an animal specimen,
11. Marie _________ by leaving Poland and traveling to scientists have concluded that the eruption changed large
France to enter the Sorbonne. portions of the area's geography. For instance, it turned the Sarno
River from its course and raised the level of the beach along the
A. challenged authority Bay of Naples. Meteorologists studying these events have also
B. showed intelligence concluded that Vesuvius caused a huge tidal wave that affected
C. behaved the world's climate.
D. was distressed
E. Answer not available In addition to making these investigations, archaeologists have
been able to study the skeletons of victims by using distilled
12. _________ she remembered their joy together. water to wash away the volcanic ash. By strengthening the brittle
bones with acrylic paint, scientists have been able to examine the
skeletons and draw conclusions about the diet and habits of the
residents. Finally, the excavations at both Pompeii and dramatic success, adding gold and silver to England's treasury
Herculaneum have yielded many examples of classical art, such and diminishing Spain's supremacy.
as jewelry made of bronze, which is an alloy of copper and tin.
The eruption of Mount Vesuvius and its tragic consequences Religious differences also caused conflict between the two
have provided everyone with a wealth of data about the effects countries. Whereas Spain was Roman Catholic, most of England
that volcanoes can have on the surrounding area. Today, had become Protestant. King Philip II of Spain wanted to claim
volcanologists can locate and predict eruptions, saving lives and the throne and make England a Catholic country again. To
preventing the destruction of other cities and cultures. satisfy his ambition and also to retaliate against England's theft
of his gold and silver, King Philip began to build his fleet of
15. Herculaneum and its harbor were buried under warships, the Spanish Armada, in January 1586.
_________ lava.
Philip intended his fleet to be indestructible. In addition to
A. liquid building new warships, he marshaled 130 sailing vessels of all
B. solid types and recruited more than 19,000 robust soldiers and 8,000
C. flowing sailors. Although some of his ships lacked guns and others
D. gas lacked ammunition, Philip was convinced that his Armada could
E. Answer not available withstand any battle with England.
16. The poisonous gases were not _________ in the air. The martial Armada set sail from Lisbon, Portugal, on May 9,
1588, but bad weather forced it back to port. The voyage
A. able to float resumed on July 22 after the weather became more stable.
B. visible
C. able to evaporate The Spanish fleet met the smaller, faster, and more
D. invisible maneuverable English ships in battle off the coast of Plymouth,
E. able to condense England, first on July 31 and again on August 2. The two battles
left Spain vulnerable, having lost several ships and with its
17. Scientists analyzed data about Vesuvius in the same way ammunition depleted. On August 7, while the Armada lay at
that a zoologist _________ a specimen. anchor on the French side of the Strait of Dover, England sent
eight burning ships into the midst of the Spanish fleet to set it on
A. describes in detail fire. Blocked on one side, the Spanish ships could only drift
B. studies by cutting apart away, their crews in panic and disorder. Before the Armada
could regroup, the English attacked again on August 8.
C. photographs
D. chart
E. Answer not available Although the Spaniards made a valiant effort to fight back, the
fleet suffered extensive damage. During the eight hours of battle,
the Armada drifted perilously close to the rocky coastline. At the
18. _________ have concluded that the volcanic eruption
moment when it seemed that the Spanish ships would be driven
caused a tidal wave.
onto the English shore, the wind shifted, and the Armada drifted
out into the North Sea. The Spaniards recognized the superiority
A. Scientists who study oceans of the English fleet and returned home, defeated.
B. Scientists who study atmospheric conditions
C. Scientists who study ash
D. Scientists who study animal behavior 20. Sir Francis Drake added wealth to the treasury and
E. Answer not available in article diminished Spain's _________.
A. unlimited power
19. Scientists have used _________ water to wash away
B. unrestricted growth
volcanic ash from the skeletons of victims.
C. territory
D. treaties
A. bottled E. Answer not available in article
B. volcanic
C. purified
D. sea 21. King Philip recruited many ______ soldiers and sailors.
E. fountain
A. warlike
The following passage refers to questions 20-24. B. strong
C. accomplished
D. timid
Conflict had existed between Spain and England since the 1570s. E. inexperienced
England wanted a share of the wealth that Spain had been taking
from the lands it had claimed in the Americas.
22. The ______ Armada set sail on May 9, 1588.
Elizabeth I, Queen of England, encouraged her staunch admiral
A. complete
of the navy, Sir Francis Drake, to raid Spanish ships and towns.
B. warlike
Though these raids were on a small scale, Drake achieved
C. independent C. wanted to fight
D. isolated D. given orders for all to fight
E. Answer not available E. defeated
23. The two battles left the Spanish fleet _________. 26. Darius took drastic steps to ________ the rebellious
Athenians.
A. open to change
B. triumphant A. weaken
C. open to attack B. destroy
D. defeated C. calm
E. discouraged D. irritate
E. Answer not available
24. The Armada was ______ on one side.
27. Their participation _________ to the Athenians.
A. closed off
B. damaged A. gave comfort
C. alone B. gave honor
D. circled C. gave strength
E. Answer not available in this article D. gave fear
E. gave hope
The following passage refers to questions 25-29.
28. The people of Delos did not want to ______ the conquest
The victory of the small Greek democracy of Athens over the of Greece.
mighty Persian Empire in 490 B.C. is one of the most famous
events in history. Darius, king of the Persian Empire, was furious A. end
because Athens had interceded for the other Greek city-states in B. encourage
revolt against Persian domination. In anger the king sent an C. think about
enormous army to defeat Athens. He thought it would take D. daydream about
drastic steps to pacify the rebellious part of the empire. E. Answer not available
Persia was ruled by one man. In Athens, however, all citizens 29. The Athenians were _________ by some soldiers who
helped to rule. Ennobled by this participation, Athenians were arrived from Plataea.
prepared to die for their city-state. Perhaps this was the secret of
the remarkable victory at Marathon, which freed them from A. welcomed
Persian rule. On their way to Marathon, the Persians tried to fool B. strengthened
some Greek city-states by claiming to have come in peace. The C. held
frightened citizens of Delos refused to believe this. Not wanting D. captured
to abet the conquest of Greece, they fled from their city and did E. Answer not available
not return until the Persians had left. They were wise, for the
Persians next conquered the city of Eritrea and captured its The following passage refers to questions 30-32.
people.
The Trojan War is one of the most famous wars in history. It is
Tiny Athens stood alone against Persia. The Athenian people
well known for the 10-year duration, for the heroism of a number
went to their sanctuaries. There they prayed for deliverance. of legendary characters, and for the Trojan horse. What may not
They asked their gods to expedite their victory. The Athenians be familiar, however, is the story of how the war began.
refurbished their weapons and moved to the plain of Marathon,
where their little band would meet the Persians. At the last
moment, soldiers from Plataea reinforced the Athenian troops. According to Greek myth, the strife between the Trojans and the
Greeks started at the wedding of Peleus, King of Thessaly, and
Thetis, a sea nymph. All of the gods and goddesses had been
The Athenian army attacked, and Greek citizens fought bravely. invited to the wedding celebration in Troy except Eris, goddess
The power of the mighty Persians was offset by the love that the
of discord. She had been omitted from the guest list because her
Athenians had for their city. Athenians defeated the Persians in
presence always embroiled mortals and immortals alike in
both archery and hand combat. Greek soldiers seized Persian
conflict.
ships and burned them, and the Persians fled in terror.
Herodotus, a famous historian, reports that 6,400 Persians died,
compared to only 192 Athenians. To take revenge on those who had slighted her, Eris decided to
cause a skirmish. Into the middle of the banquet hall, she threw a
golden apple marked "for the most beautiful." All of the
25. Athens had _________ the other Greek city-states against goddesses began to haggle over who should possess it. The gods
the Persians. and goddesses reached a stalemate when the choice was
narrowed to Hera, Athena, and Aphrodite. Someone was needed
A. refused help to to settle the controversy by picking a winner. The job eventually
B. intervened on behalf of fell to Paris, son of King Priam of Troy, who was said to be a
good judge of beauty. Paris did not have an easy job. Each In 1920, 18 months after the czar's execution, this terrified young
goddess, eager to win the golden apple, tried aggressively to woman was rescued from drowning in a Berlin river. She spent
bribe him. two years in a hospital, where she attempted to reclaim her
health and shattered mind. The doctors and nurses thought that
"I'll grant you vast kingdoms to rule," promised Hera. "Vast she resembled Anastasia and questioned her about her
kingdoms are nothing in comparison with my gift," contradicted background. She disclaimed any connection with the czar's
Athena. "Choose me and I'll see that you win victory and fame in family. Eight years later, however, she claimed that she was
war." Aphrodite outdid her adversaries, however. She won the Anastasia. She said that she had been rescued by two Russian
golden apple by offering Helen, daughter of Zeus and the most soldiers after the czar and the rest of her family had been killed.
beautiful mortal in the land, to Paris. Paris, anxious to claim Two brothers named Tschaikovsky had carried her into
Helen, set off for Sparta in Greece. Romania. She had married one of the brothers, who had taken
her to Berlin and left her there, penniless and without a vocation.
Unable to invoke the aid of her mother's family in Germany, she
Although Paris learned that Helen was married, he nevertheless
had tried to drown herself.
accepted the hospitality of her husband, King Menelaus of
Sparta. Therefore, Menelaus was outraged for a number of
reasons when Paris departed, taking Helen and much of the During the next few years, scores of the czar's relatives, ex-
king's wealth back to Troy. Menelaus collected his loyal forces servants, and acquaintances interviewed her. Many of these
and set sail for Troy to begin the war to reclaim Helen. people said that her looks and mannerisms were evocative of the
Anastasia that they had known. Her grandmother and other
relatives denied that she was the real Anastasia, however.
30. Eris was known for _________ both mortals and
immortals.
Tired of being accused of fraud, Anastasia immigrated to the
United States in 1928 and took the name Anna Anderson. She
A. scheming against
still wished to prove that she was Anastasia, though, and
B. creating conflict amongst
returned to Germany in 1933 to bring suit against her mother's
C. feeling hostile toward
D. ignoring family. There she declaimed to the court, asserting that she was
E. comforting indeed Anastasia and deserved her inheritance.
In 1957, the court decided that it could neither confirm nor deny
31. Each goddess tried ______ to bribe Paris.
Anastasia's identity. Although it will probably never be known
whether this woman was the Grand Duchess Anastasia, her
A. boldly search to establish her identity has been the subject of numerous
B. effectively books, plays, and movies.
C. secretly
D. carefully
E. Answer not available 33. Some Russian peasants and workers ______ for social
reform.
32. Athena _________ Hera, promising Paris victory and
A. longed
fame in war.
B. cried out
C. begged
A. disregarded the statement of D. hoped
B. defeated E. thought much
C. agreed with
D. restated the statement of
E. questioned the statement of 34. Witnesses ______ that all members of the czar's family
had been executed.
Refer to the following passage for questions 33-37.
A. gave assurance
B. thought
One of the most intriguing stories of the Russian Revolution C. hoped
concerns the identity of Anastasia, the youngest daughter of Czar D. convinced some
Nicholas II. During his reign over Russia, the czar had planned E. Answer not available
to revoke many of the harsh laws established by previous czars.
Some workers and peasants, however, clamored for more rapid
social reform. In 1918, a group of these people known as 35. Tschaikovsky initially ______ any connection with the
Bolsheviks overthrew the government. On July 17 or 18, they czar's family.
murdered the czar and what was thought to be his entire family.
A. denied
B. stopped
Although witnesses vouched that all the members of the czar's
C. noted
family had been executed, there were rumors suggesting that
D. justified
Anastasia had survived. Over the years, a number of women
claimed to be Grand Duchess Anastasia. Perhaps the most E. Answer not available
famous claimant was Anastasia Tschaikovsky, who was also
known as Anna Anderson. 36. She was unable to ______ the aid of her relatives.
A. locate D. individuals
B. speak about E. spenders
C. call upon
D. identify Refer to the following passage for questions 40-45.
E. know
Many great inventions are initially greeted with ridicule and
37. In court she _________ maintaining that she was disbelief. The invention of the airplane was no exception.
Anastasia and deserved her inheritance. Although many people who heard about the first powered flight
on December 17, 1903 were excited and impressed, others
A. finally appeared reacted with peals of laughter. The idea of flying an aircraft was
B. spoke forcefully repulsive to some people. Such people called Wilbur and Orville
C. gave testimony Wright, the inventors of the first flying machine, impulsive fools.
D. gave evidence Negative reactions, however, did not stop the Wrights. Impelled
E. Answer not available by their desire to succeed, they continued their experiments in
aviation.
Refer to the following passage for questions 38-39.
Orville and Wilbur Wright had always had a compelling interest
King Louis XVI and Queen Marie Antoinette ruled France from in aeronautics and mechanics. As young boys they earned money
1774 to 1789, a time when the country was fighting bankruptcy. by making and selling kites and mechanical toys. Later, they
The royal couple did not let France's insecure financial situation designed a newspaper-folding machine, built a printing press,
limit their immoderate spending, however. Even though the and operated a bicycle-repair shop. In 1896, when they read
minister of finance repeatedly warned the king and queen against about the death of Otto Lilienthal, the brothers' interest in flight
wasting money, they continued to spend great fortunes on their grew into a compulsion.
personal pleasure. This lavish spending greatly enraged the
people of France. They felt that the royal couple bought its Lilienthal, a pioneer in hang-gliding, had controlled his gliders
luxurious lifestyle at the poor people's expense. by shifting his body in the desired direction. This idea was
repellent to the Wright brothers, however, and they searched for
Marie Antoinette, the beautiful but exceedingly impractical more efficient methods to control the balance of airborne
queen, seemed uncaring about her subjects' misery. While vehicles. In 1900 and 1901, the Wrights tested numerous gliders
French citizens begged for lower taxes, the queen embellished and developed control techniques. The brothers' inability to
her palace with extravagant works of art. She also surrounded obtain enough lift power for the gliders almost led them to
herself with artists, writers, and musicians, who encouraged the abandon their efforts.
queen to spend money even more profusely.
After further study, the Wright brothers concluded that the
While the queen's favorites glutted themselves on huge feasts at published tables of air pressure on curved surfaces must be
the royal table, many people in France were starving. The French wrong. They set up a wind tunnel and began a series of
government taxed the citizens outrageously. These high taxes experiments with model wings. Because of their efforts, the old
paid for the entertainments the queen and her court so enjoyed. tables were repealed in time and replaced by the first reliable
When the minister of finance tried to stop these royal figures for air pressure on curved surfaces. This work, in turn,
spendthrifts, the queen replaced him. The intense hatred that the made it possible for the brothers to design a machine that would
people felt for Louis XVI and Marie Antoinette kept building fly. In 1903 the Wrights built their first airplane, which cost less
until it led to the French Revolution. During this time of struggle than $1,000. They even designed and built their own source of
and violence (1789-1799), thousands of aristocrats, as well as the propulsion-a lightweight gasoline engine. When they started the
king and queen themselves, lost their lives at the guillotine. engine on December 17, the airplane pulsated wildly before
Perhaps if Louis XVI and Marie Antoinette had reined in their taking off. The plane managed to stay aloft for 12 seconds,
extravagant spending, the events that rocked France would not however, and it flew 120 feet.
have occurred.
By 1905, the Wrights had perfected the first airplane that could
38. The people surrounding the queen encouraged her to turn, circle, and remain airborne for half an hour at a time.
spend money ______. Others had flown in balloons and hang gliders, but the Wright
brothers were the first to build a full-size machine that could fly
A. wisely under its own power. As the contributors of one of the most
outstanding engineering achievements in history, the Wright
B. abundantly
brothers are accurately called the fathers of aviation.
C. carefully
D. foolishly
E. joyfully 40. The idea of flying an aircraft was ______ to some people.
39. The minister of finance tried to curb these royal ______. A. boring
B. distasteful
C. exciting
A. aristocrats
B. money wasters D. needless
C. enemies E. Answer not available
41. People thought that the Wright brothers had ______. 5. C: A peninsula is a piece of land connected to the mainland by
an isthmus and projecting into the ocean such that it is
A. acted without thinking surrounded on three sides by water. A peninsula is not a coast
B. been negatively influenced (A); it is not found inland (B); and it is not a border (D).
C. been too cautious
D. been mistaken 6. B: The passage was found near 50 degrees S latitude.
E. acted in a negative way Latitudes are measured horizontally, in relation to the equator or
central imaginary line, equidistant between the North and South
42. The Wrights' interest in flight grew into a ______. Poles. Longitudes are measured vertically. Greenwich (A), the
location of zero degrees longitude, adopted as the global
A. financial empire standard, is both incorrect and never named in the passage. Spain
(C), Portugal (D), and Madrid (E) in Spain are also incorrect.
B. plan
C. need to act
D. foolish thought 7. A: Meridians are imaginary geographical circles intersecting
E. Answer not available the poles. Imaginary lines parallel to the equator (B) are
latitudes. The International Date Line is a specific meridian, not
an area (C). It is not a land mass (D) as it crosses both water and
43. Lilienthal's idea about controlling airborne vehicles was
land.
_________ the Wrights.
1. B: "Terrestrial" means land. No choice here offers a synonym 13. C: The closest synonym for the "feeling of desolation"
for "marine," e.g. nautical/naval/water/seagoing, and no other (despair) described in the passage is wretchedness. Misfortune
choices match either marine or terrestrial. (A) or ill fate/luck is not as close. Anger (B) is a separate
emotion from desolation. Disappointment (D) is also different
2. A: "Quagmire" means literally a bog or marsh, and from desolation, meaning feeling let-down rather than hopeless.
figuratively an involved situation difficult to escape; Ambition (E) is drive to succeed or accomplish things. It was not
entanglement is a synonym, more specifically similar than the Curie's ambition that faded upon returning to the Sorbonne but
other choices. her depression.
3. A: Longitudes are imaginary geographical lines running north 14. C: "Disillusioned" means disappointed. It does not mean
and south. Latitudes run east and west. The other choices do not troubled (A), i.e. concerned or disturbed; worried (B) or anxious;
equal either latitude or longitude in direction. sorrowful (D) or sad; or disturbed (E).
4. C: Topography means the physical features of a land mass. It 15. B: "Coagulated" means solidified. Liquid (A) is an opposite
does not mean coastline (A), mountain range (B), or islands (D). of solid. Flowing (C) assumes a liquid, not solid, state. Gas (D)
is another opposite of solid. (Three states of matter, like volcanic fortified or reinforced, gave fear (D) or frightened, or gave hope
material, are liquid, solid, and gaseous.) (E) or encouraged.
16. A: "Buoyant" means able to float. The passage indicates this 28. B: To "abet" means to enable, support, or encourage, usually
by indicating that the gases therefore, sank toward earth and in crime or doing something wrong. It does not mean to end (A),
suffocated people. Buoyant does not mean visible (B) or possible think about (C), or daydream about (D) something.
to see. Able to float/buoyant does not mean able to evaporate
(C). Evaporation means turning to vapor, which only liquids can 29. B: "Reinforced" means strengthened, not welcomed (A), held
do. Gases are already vapors. Buoyant does not mean invisible (C), or captured (D).
(D) or unseen. Able to float does not mean able to condense (E),
i.e. turn from vapor to liquid. 30. B: The passage states that the presence of Eris, goddess of
discord, "always embroiled mortals and immortals alike in
17. B: "Dissect" means to cut apart for study. It does not mean to conflict." Embroiling them in conflict is creating conflict
describe in detail (A), to photograph (C), or to chart (D) a amongst them. It does not mean scheming against (A) them,
specimen. feeling hostile toward (C) them, ignoring (D) them, or
comforting (E) them.
18. B: Meteorologists are scientists who study atmospheric
conditions, particularly weather. Scientists who study oceans (A) 31. A: "Aggressively" means boldly. It does not mean
are oceanographers, i.e. marine scientists. Scientists who study effectively (B) or successfully, secretly (C), or carefully (D).
ash (C) do not exist as members of a separate discipline. Climate
scientists and many others concerned with its effects study 32. A: "Contradicted" means Athena disregarded Hera's
volcanic ash. Scientists who study animal behavior (D) are
statement and disputed or countered it. It does not mean she
ethologists or animal behaviorists and do not study ash.
defeated (B) her statement, agreed with (C) it, restated (D) it, or
questioned (E) it.
19. C: Distilled water is purified water. Distilled water is not
equivalent to bottled (A), volcanic (B), sea (D), or fountain (E)
33. B: To "clamor for" means to cry out for (something). It does
water.
not mean to long for (A) it, beg (C) for it, hope (D) for it, or
think much (E) "for," of, or about it.
20. A: "Supremacy" means unlimited power, not unrestricted
growth (B). The passage states that Drake diminished Spain's 34. A: To "vouch" means to give assurance. It does not mean to
supremacy, but does not specifically mention diminishing its
think (B), hope (C), or convince some (D).
territory (C). Drake's raids enriched England and reduced Spain's
power; no mention is made of eliminating any treaties (D).
35. A: "Disclaimed" means denied, i.e. refused or declared
untrue. It does not mean stopped (B), noted (C), or justified (D),
21. B: "Robust" means strong. It does not mean warlike (A), i.e. substantiated or confirmed, the opposite of denied.
accomplished (C) or competent, timid (D) or fearful, or
inexperienced (E).
36. C: She was unable to invoke, i.e. to call upon, the aid of
relatives. To invoke does not mean to locate (A) or find; to speak
22. B: "Martial" means warlike or war-related. It does not mean about (B) or discuss; to identify (D), i.e. recognize; or to know
complete (A), independent (C), or isolated (D). (E).
When John was 11 years old, his family moved to the United A. Between 1901 and 1906
States and settled in Wisconsin. John was good with tools and B. Between 1838 and 1868
soon became an inventor. He first invented a model of a sawmill. C. Between 1906 and 1914
Later, he invented an alarm clock that would cause the sleeping D. Between 1868 and 1901
person to be tipped out of bed when the timer sounded. E. Between 1906 and 1907
Muir left home at an early age. He took a 1,000-mile walk south 5. What happened last?
to the Gulf of Mexico in 1867and 1868. Then he sailed for San
Francisco. The city was too noisy and crowded for Muir, so he A. John Muir died.
headed inland for the Sierra Nevadas. B. John Muir Trail was dedicated.
C. Muir's glacial theory was proven.
When Muir discovered the Yosemite Valley in the Sierra D. The Sierra Club was formed.
Nevadas, it was as if he had come home. He loved the E. John's family visited him.
mountains, the wildlife, and the trees. He climbed the mountains
and even climbed trees during thunderstorms in order to get Refer to the following passage for questions 6 through 9.
closer to the wind. He put forth the theory in the late 1860s that
the Yosemite Valley had been formed through the action of When using a metal file, always remember to bear down on the
glaciers. People ridiculed him. Not until 1930 was Muir's theory forward stroke only. On the return stroke, lift the file clear of the
proven correct. surface to avoid dulling the instrument's teeth. Only when
working on very soft metals is it advisable to drag the file's teeth
slightly on the return stroke. This helps clear out metal pieces hunger. She herself had been sold several times. By 1827, when
from between the teeth. New York freed its slaves, she had married and given birth to
four children.
It is best to bear down just hard enough to keep the file cutting at
all times. Too little pressure uses only the tips of the teeth, while The first hint of Isabella's fighting spirit came soon afterwards,
too much pressure can chip the teeth. Move the file in straight when her youngest son was illegally seized and sold. She
lines across the surface. Use a vise to grip the work so that your marched to the courthouse and badgered officials until her son
hands are free to hold the file. Protect your hands by equipping was returned to her.
the file with a handle. Buy a wooden handle and install it by
inserting the pointed end of the file into the handle hole. In 1843, inspired by religion, she changed her name to Sojourner
(meaning "one who stays briefly") Truth and, with only pennies
6. These directions show you how to... in her purse, set out to preach against slavery. From New
England to Minnesota she trekked, gaining a reputation for her
A. Work with a hammer. plain but powerful and moving words. Incredibly, despite being
B. Use a file. black and female (only white males were expected to be public
C. Polish a file. speakers), she drew thousands to town halls, tents, and churches
D. Oil a vise. to hear her powerful, deep-voiced pleas on equality for blacks-
E. Repair shop tools. and for women. Often she had to face threatening hoodlums.
Once she stood before armed bullies and sang a hymn to them.
Awed by her courage and her commanding presence, they
7. When using a file...
sheepishly retreated.
A. Always bear down on the return stroke.
During the Civil War she cared for homeless ex-slaves in
B. Move it in a circle.
Washington, D.C. President Lincoln invited her to the White
C. Remove the handle.
House to bestow praise on her. Later, she petitioned Congress to
D. Press down on the forward stroke.
E. Wear protective gloves. help former slaves get land in the West. Even in her old age, she
forced the city of Washington, D.C. to integrate its trolley cars so
that black and white passengers could ride together.
8. When working on soft metals, you can...
Shortly before her death at the age of 86, she was asked what
A. Remove the handle. kept her going. "I think of the great things," replied Sojourner.
B. Clear metal pieces from the teeth.
C. Bear down very hard on the return stroke.
D. File in circles. 10. The imposing black woman promised to keep the white
E. Strengthen them with added wood. man...
A. Searching.
9. Protect your hands by...
B. Crying.
C. Hollering.
A. Dulling the teeth. D. Scratching.
B. Dragging the teeth on the backstroke. E. Fleeing.
C. Using a vise.
D. Installing a handle.
E. Wearing safety gloves. 11. This incident occurred in the...
15. Her change in name was inspired by... Ships would anchor off the islands, and crews would row ashore
and seize as many tortoises as they could. Once the animals were
aboard the ship, the sailors would roll the tortoises onto their
A. A fighting spirit.
B. Religion. backs. The tortoises were completely helpless once on their
C. Her freedom. backs, so they could only lie there until used for soups and stews.
Almost 100,000 tortoises were carried off in this way.
D. Officials.
E. Friends.
The tortoises faced other problems, too. Soon after the first
ships, settlers arrived, bringing pigs, goats, donkeys, dogs and
16. She traveled from New England to...
cats. All of these animals ruined life for the tortoises. Donkeys
and goats ate all the plants that the tortoises usually fed on, while
A. Canada. the pigs, dogs and cats consumed thousands of baby tortoises
B. California. each year. Within a few years, it was hard to find any tortoise
C. Minnesota. eggs-or even any baby tortoises.
D. Alaska.
E. Virginia.
By the early 1900s, people began to worry that the last of the
tortoises would soon die out. No one, however, seemed to care
17. She forced the city of Washington, D.C. to... enough to do anything about the problem. More and more
tortoises disappeared, even though sailors no longer needed them
A. Integrate its trolleys. for food. For another 50 years, this situation continued. Finally,
B. Give land grants. in the 1950s, scientists decided that something must be done.
C. Care for ex-slaves.
D. Provide food for ex-slaves The first part of their plan was to remove as many cats, dogs and
. E. Clean its trolleys. other animals as they could from the islands. Next, they tried to
make sure that more baby tortoises would be born. To do this,
18. She preached against... they started looking for wild tortoise eggs. They gathered the
eggs and put them in safe containers. When the eggs hatched, the
A. Smoking. scientists raised the tortoises in special pens. Both the eggs and
B. Slavery. tortoises were numbered so that the scientists knew exactly
C. Alcohol. which kinds of tortoises they had and which island they came
D. Hoodlums. from. Once the tortoises were old enough and big enough to take
E. Women having no rights. care of themselves, the scientists took them back to their islands
and set them loose. This slow, hard work continues today, and,
19. Sojourner Truth died at... thanks to it, the number of tortoises is now increasing every year.
Perhaps these wonderful animals will not disappear after all.
A. 48.
B. 72. 20. What happened first?
C. 63.
D. 86. A. Sailors took tortoises aboard ships.
E. 88. B. The tortoise meat was used for soups and stews.
C. Tortoises were put onto their backs.
Refer to the following passage for questions 20 through 24. D. Settlers brought other animals to the islands.
E. Pigs had been all the sailors had to eat.
The Galapagos Islands are in the Pacific Ocean, off the western
coast of South America. They are a rocky, lonely spot, but they 21. What happened soon after people brought animals to the
are also one of the most unusual places in the world. One reason islands?
is that they are the home of some of the last giant tortoises left on
earth. A. Tortoise eggs were kept in safe containers.
B. Scientists took away as many animals as they could.
Weighing hundreds of pounds, these tortoises, or land turtles, C. The animals ate the tortoises' food and eggs.
wander slowly around the rocks and sand of the islands. D. The tortoises fought with the other animals.
E. The tortoises continued to wander freely.
22. When did people start to do something to save the 28. The answer must be...
tortoises?
A. In New York.
A. In the 1500s B. Within the United States.
B. In the 1950s C. A proper geographical term.
C. In the early 1900s D. In the same region.
D. In the 1960s E. Along a coast line.
E. In the 1400s
Refer to the following passage for questions 29 through 33.
23. What happens right after the tortoise eggs hatch?
Charles A. Lindbergh is remembered as the first person to make
A. The scientists take the tortoises back to their islands. a nonstop solo flight across the Atlantic, in 1927. This feat,
B. The scientists get rid of cats, dogs, and other animals. performed when Lindbergh was only 25 years old, assured him a
C. The sailors use the tortoises for food. lifetime of fame and public attention.
D. The scientists raise the tortoises in special pens.
E. The scientists encourage the villagers to help. Charles Augustus Lindbergh was more interested in flying
airplanes than he was in studying. He dropped out of the
24. What happened last? University of Wisconsin after two years to earn a living
performing daredevil airplane stunts at county fairs. Two years
A. The tortoises began to disappear. later, he joined the United States Army so that he could go to the
B. The number of tortoises began to grow. Army Air Service flight-training school. After completing his
C. Scientists took away other animals. training, he was hired to fly mail between St. Louis and Chicago.
D. Tortoises were taken back to their home islands.
E. The number of tortoises began to decrease. Then came the historic flight across the Atlantic. In 1919, a New
York City hotel owner offered a prize of $25,000 for the first
Refer to the following passage for questions 25 through 28. pilot to fly nonstop from New York to Paris. Nine St. Louis
business leaders helped pay for the plane Lindbergh designed
The first person in the group starts off by naming anything that is especially for the flight. Lindbergh tested the plane by flying it
geographical. It could be a city, state, country, river, lake, or any from San Diego to New York, with an overnight stop in St.
proper geographical term. For example, the person might say, Louis. The flight took only 20 hours and 21 minutes, a
"Boston." The second person has 10 seconds to think of how the transcontinental record.
word ends and come up with another geographical term starting
with that letter. The second participant might say, "Norway," Nine days later, on May 20, 1927, Lindbergh took off from Long
because the geographical term has to start with "N." The third Island, New York, at 7:52 a.m. He landed in Paris on May 21 at
person would have to choose a word beginning with "Y." If a 10:21 p.m. He had flown more than 3,600 miles in less than 34
player fails to think of a correct answer within the time limit, that hours. His flight made news around the world. He was given
player is out of the game. The last person to survive is the awards and parades everywhere he went. He was presented with
champion. the US Congressional Medal of Honor and the first
Distinguished Flying Cross. For a long time, Lindbergh toured
25. This game may help you with... the world as a US goodwill ambassador. He met his future wife,
Anne Morrow, in Mexico, where her father was the United
States ambassador.
A. History.
B. Music.
C. Geography. During the 1930s, Charles and Anne Lindbergh worked for
D. Sports. various airline companies, charting new commercial air routes.
E. Current events. In 1931, for a major airline, they charted a new route from the
east coast of the United States to the Orient. The shortest, most
efficient route was a great curve across Canada, over Alaska, and
26. The person trying to answer needs...
down to China and Japan. Most pilots familiar with the Arctic
did not believe that such a route was possible. The Lindberghs
A. No time limit. took on the task of proving that it was. They arranged for fuel
B. To know geography only. and supplies to be set out along the route. On July 29, they took
C. To ignore the last letters of words. off from Long Island in a specially equipped small seaplane.
D. To know something about spelling and geography. They flew by day and each night landed on a lake or a river and
E. To be a good speller. camped. Near Nome, Alaska, they had their first serious
emergency. Out of daylight and nearly out of fuel, they were
27. Before you choose your own word, think about how... forced down into a small ocean inlet. In the next morning's light,
they discovered they had landed on barely three feet of water. On
A. The last word starts. September 19, after two more emergency landings and numerous
B. The last word ends. close calls, they landed in China with the maps for a safe airline
C. Smart you are. passenger route.
D. Long the last word is.
E. The first word is spelled.
Even while actively engaged as a pioneering flier, Lindbergh record the figure it has just passed. Continue recording the
was also working as an engineer. In 1935, he and Dr. Alexis figures on the dials from right to left. When finished, mark off
Carrel were given a patent for an artificial heart. During World the number of units recorded. Dials on water and gas meters
War II in the 1940s, Lindbergh served as a civilian technical usually indicate the amount each dial records.
advisor in aviation. Although he was a civilian, he flew over 50
combat missions in the Pacific. In the 1950s, Lindbergh helped 34. These instructions show you how to...
design the famous 747 jet airliner. In the late 1960s, he spoke
widely on conservation issues. He died in August 1974, having
A. Read a meter.
lived through aviation history from the time of the first powered
B. Turn the dials of a meter.
flight to the first steps on the moon and having influenced a big
C. Install a gas meter.
part of that history himself. D. Repair a water meter.
E. Be prepared for outside employment.
29. What did Lindbergh do before he crossed the Atlantic?
35. Always read the meter dials...
A. He charted a route to China.
B. He graduated from flight-training school. A. From top to bottom.
C. He married Anne Morrow.
B. From right to left.
D. He acted as a technical advisor during World War II.
C. From left to right.
E. He was responsible for the fuel supply for planes.
D. From the small to the large dial.
E. From the large dial to the small dial.
30. What happened immediately after Lindbergh crossed the
Atlantic?
36. As you read the first dial, record the figures...
31. When did Charles meet Anne Morrow? 37. When you have finished reading the meter, mark off...
32. When did the Lindberghs map an air route to China? Refer to the following passage for questions 38 through 44.
9. D: The instructions advise users to install a handle to protect 19. D: The last paragraph of the passage informs that Sojourner
their hands rather than dulling the teeth (A), against which they Truth died at the age of 86, not at 48 (A), 72 (B), 63 (C), or 88
advise; dragging the teeth on the return stroke (B), which is (E) years.
recommended NOT for protecting hands but for clearing the
file's teeth of pieces from very soft metals; using a vise (C), 20. A: The fourth paragraph describes sailors' carrying tortoises
which is recommended to free the hands, not protect them; or off to their ships, where they subsequently turned them onto their
wearing safety gloves (E), which is never mentioned. backs (C), rendering them helpless, and then used them for food
(B). Settlers brought other animals to the Galapagos Islands (D)
10. D: The second paragraph quotes Truth as saying, "I'll keep "soon after" (fifth paragraph) the sailors took them. The passage
you scratching" in response to the white man's comparison of her never mentions the sailors' eating pigs (E) at all.
speech to a flea's bite.
21. C: The passage describes (fifth paragraph) animals' eating
11. C: The third paragraph places this incident in the 1840s. the tortoises' food and eggs. Tortoise eggs were kept in safe
According to the passage's information, Sojourner Truth was not containers (A) many years later, in the 1950s (penultimate and
yet born in the 1760s (A) or 1700s (E). It also states she died at last paragraphs), once scientists began to rebuild the depleted
age 86, so she was not still alive in the 1900s (B) or 1920s (D). tortoise population. Scientists gathered tortoise eggs but did not
(Note: Sojourner Truth lived 1797-1883. The passage does not take away tortoises (B); sailors many years earlier did. The
give these specific years, but the wrong answers can be tortoises did not fight with other animals (D) or continue to
identified through the information it does give, described above.) wander freely (E), as more and more disappeared.
12. A: The passage identifies upstate New York as where 22. B: The passage indicates (sixth and seventh/last paragraphs)
Sojourner Truth was raised (fourth paragraph). It never mentions that scientists started working to save tortoises in the 1950s. The
Georgia (B), New Jersey (C), Idaho (D), or Maryland (E). It 1500s (A) was when sailors first visited the Galapagos and began
mentions her trekking from New England to Minnesota (sixth decimating the tortoise population by eating them. The early
paragraph) preaching against slavery; and caring for homeless 1900s (C) is described as when people began to worry about
ex-slaves during the Civil War and forcing the city to integrate tortoise extinction, not when scientists began to do something
trolley cars in her old age, both in Washington, D.C.; but not about it. The 1960s (D) and 1400s (E) are never mentioned.
growing up anywhere other than New York State.
23. D: The passage describes scientists raising newly hatched
13. D: The passage indicates fourth paragraph) that Isabella had tortoise eggs in special pens. They only return them to their
lost both parents by the time she was 14 years old, not 27 (A), 2 islands (A) once they have grown old and big enough to care for
(B), 7 (C), or 19 (E) years old. themselves. The scientists got rid of excessive cats, dogs, and
other animals (B) to decrease their predation on tortoises before
14. C: The fourth paragraph of the passage indicates that Isabella gathering, incubating, and hatching tortoise eggs. Sailors first
had married and had four children by the time New York freed used tortoises for food (C) hundreds of years ago. The passage
its slaves in 1827. It does not indicate that she had problems (A), never mentions their encouraging villagers to help (E).
no children (B), an education (D), or three children (E) by that
time. 24. B: Tortoise numbers began growing thanks to the scientists'
efforts. The number of tortoises began to decrease (E) and
15. B: The sixth paragraph indicates that Isabella, "inspired by tortoises began disappearing (A) first. Years later, scientists
religion," changed her name to Sojourner Truth in 1843. The removed other animals (C) that had been preying on tortoises.
previous (fifth) paragraph refers to her fighting spirit (A) as After gathering and hatching eggs and raising tortoises, scientists
signified by her demanding her son's return, not as the returned them to their islands (D) and tortoise numbers began
inspiration for her name change. The passage does not indicate increasing.
that she was inspired by her freedom (C), by officials (D), or by
friends (E) to change her name. 25. C: This game can help the players learn geography through
naming geographical terms and names. This activity will not
16. C: The sixth paragraph describes Truth's traveling from New help players learn history (A), music (B), sports (D), or current
England to Minnesota, not Canada (A), California (B), Alaska events (E).
(D), or Virginia (E).
26. D: The person answering needs to know geographical names
and terms, and how to spell them. The game does give each
player a time limit (A) of 10 seconds to answer. The game does 35. B: The first sentence advises always to read meter dials from
not require players to know only geography (B). The game right to left, not from top to bottom (A), left to right (C), from
requires attending to, not ignoring (C) the last letters of words. the small to large dial (D), or from the large to small dial.
While players need to know some spelling, with familiar place-
names such as the examples given they need not be especially 36. C: The instructions indicate always to record the figure the
good spellers (E). pointer has just passed, not the figures it is approaching (B), as it
turns clockwise. They never instruct to record the smaller dial's
27. B: How the last word ends enables each player to think of a figures (A); in fact, they stipulate that it is not imperative even to
name/word starting with that letter. How the previous word starts read the smaller dial. There are no instructions to record figures
(A) is immaterial, as is thinking about how smart you are (C). at the top (D) or bottom (E).
How long the last word is (D) does not matter. How the initial
word is spelled (E) overall is irrelevant to subsequent players' 37. A: The second-to-last sentence advises to mark off the
word choices-except the second player, who must know the first number of units recorded, not the small dial's figures (B)-which
word's final letter, but not the rest of the word's spelling. are not imperative even to read, let alone marking off these. The
instructions never advise to mark off the total figures (C), all the
28. C: Proper geographical terms are the required answers. The zero marks (D), or the last reading of the month (E).
passage never stipulates that answers must be in New York (A),
within the United States (B), in the same region (D), or along a 38. C: Heimaey is the island where the village of
coast line (E). Vestmannaeyjar (A) is located. The Hebrides (B) are islands off
the west coast of Scotland, not Iceland. Helgafell (D) is the name
29. B: Lindbergh was hired to fly mail after attending Army of the volcano that erupted in Vestmannaeyjar on Heimaey
flight-training school, subsequently crossing the Atlantic in 1919 Island. Heima (E) means "home" or "at home" in Icelandic, and
(second paragraph). He married Anne Morrow (C) after flying is the title of a documentary released in 2007 about the 2006 tour
from New York to Paris in 1927 (third paragraph). They charted of Icelandic band Sigur R�s.
the China route (A) in 1931 (fourth paragraph). The Lindberghs
arranged for fuel provision along this route; Lindbergh was not 39. D: The passage describes the lava as "red-hot liquid" in the
responsible for planes' fuel supply (E). He was a technical first paragraph. The colors orange (A), yellow (C), and gray (E)
advisor during World War II (D) in the 1940s (fifth paragraph). are not used in this passage. The cinders and ash, not the hot
liquid, are described as black (B).
30. D: Immediately after crossing the Atlantic, Lindbergh was
awarded the Congressional Medal of Honor (third paragraph). 40. B: The liquid, i.e. lava, was coming from the ground-
He flew mail between St. Louis and Chicago (A) before crossing specifically, from underground. Magma exerts pressure
the Atlantic (second paragraph). underground until it erupts on the surface as lava. Volcanoes are
mountains (A); however, lava does not come from the mountains
31. D: Charles met Anne after receiving the first Distinguished but from beneath them. The lava came from under the ground,
Flying Cross for flying from New York to Paris in 1927 (third not from the sea (C), the sky (D), or the ocean (E). (There are
paragraph). He met her after he took off from Long Island (A) to volcanoes under seas and oceans, but the one described in this
begin that flight. He and Anne both worked for airlines (B) after passage was not.)
marrying (fourth paragraph). He and Anne were forced down
into an ocean inlet (C) in Alaska while mapping the route to 41. B: The introductory paragraph indicates in its last sentence
China (fourth paragraph). The passage never mentions that the volcano had been inactive for 7,000 years, not 70 years
Lindbergh's visiting his parents (E). (A), 7,000 months (C), 700 years (D), or 70 decades (E), which
also equals 700 years (D).
32. B: The Lindberghs mapped the route to China in 1931;
Charles and Dr. Carrel received a patent for their artificial heart 42. A: The passage describes the black cinders as "baseball-size"
in 1935. The Lindberghs charted the China route as work for a (second paragraph). It does not liken their size to that of pebbles
major airline, not before doing so (A). 1931 was prior to World (B) or golf balls (C), which are both smaller than baseballs; or to
War II (C). Lindbergh helped design the 747 (D) in the 1950s. footballs (D), which are bigger than baseballs; or to hailstones
Lindbergh was 25 in 1927 (first paragraph), so he was 30 (E) in (E), which vary in size but to which the cinders are not compared
1932, a year after mapping the China route. in this passage.
33. C: Lindbergh helped design the 747 in the 1950s. The 43. D: The passage describes the street lamps still burning (third
artificial heart patent (A) was in 1935. The Lindberghs mapped a paragraph) as part of the weird aspect of the village blanketed by
route to the Orient (B) in 1931. Lindbergh flew 50 combat cinder drifts and lit by continuing fire. It never mentions buses
missions (D) during World War II in the 1940s. The passage continuing to run (A), radio broadcasts continuing (B), police
never mentions his receiving an honorary college degree (E). continuing to work (C), or television stations continuing to
broadcast (E).
34. A: These are how-to instructions for reading a meter, not for
turning a meter's dials (B) (which should not be done!), or for 44. C: The last paragraph's first sentence indicates that the
installing a gas meter (C), repairing a water meter (D), or being eruption continued for six months, not six weeks (A), six hours
prepared for outside employment (E). (B), six days (D), or six years (E).